Sei sulla pagina 1di 58

Practice Questions 1

1) A physical therapist designs a research study that examines the influence of a selected training program on
cardiovascular endurance. The therapist determines it will be necessary to utilize a control group. The PRIMARY
purpose of the control group is:
1. serve as a baseline for controlling the effects of the manipulated variable
2. function to minimize the probability of a type I error
3. eliminate the need for a systematic random sample
4. ensure reliability and validity of collected data

Answer: 1
Non-Systems
Safety & Professional Roles: Teaching/Learning Research
A control group of “placebo group” serves as a basis for experimental comparison. Specifically, the control group allows
researchers to establish a baseline for controlling the effects of the manipulated variable. (Portney p. 35)

2) A patient diagnosed with a lower extremity muscle strain is referred to physical therapy. The plan of care
incorporates ultrasound for deep heating of the involved muscle. For effective heating within a five minute period,
the area to be treated should be approximately:
1. half the size of the soundhead
2. the same size as the soundhead
3. twice the size of the soundhead
4. three times the size of the soundhead

Answer: 3
Non-Systems
Equipment & Devices; Therapeutic Modalities
Maintaining a treatment area of approximately twice the size of the soundhead allows for therapeutic heating of the tissues
using ultrasound within the established time frame. (Cameron p. 208)

3) A 55-year-old patient who sustained a left femur fracture in a motor vehicle accident is referred to physical therapy
for rehabilitation. The fracture is now healed, but the quadriceps muscle remains very weak (grade of “3” or fair).
Which activity would permit the patient to utilize the GREATEST amount of weight with the quadriceps muscle in a
sitting position?

1. eccentric isotonic exercise with a cuff weight on the ankle


2. concentric isotonic exercise with a cuff weight on the ankle
3. the patient would only be able to perform isometric exercise with a cuff weight on the ankle
4. there would be no difference in the maximum amount of weight the patient could utilize with the described
exercises

Answer: 1
Musculoskeletal System
Interventions

Scorebuilders 6
2 Practice Questions

A muscle can generate more tension with an eccentric contraction than with either an isometric or concentric exercise.
Therefore, even though the patient can only extend the knee (concentric contraction) actively with no resistance, the patient
could perform an eccentric (lengthening) contraction of the quadriceps with a small amount of weight on the ankle. (Kisner
p. 84)

4) A physical therapist recognizes the importance of providing an initial screening of risk factors and/or symptoms for
various cardiovascuflar, pulmonary and metabolic diseases to optimize patient safety during exercise programs and
testing. The Physical Activity Readiness Questionnaire (PAR-Q) has been recommended to the therapist as a
minimal standard for entry into moderate intensity exercise programs. A person who answers “yes” to any question
on the Physical Activity Readiness Questionnaire (PAR-Q):
1. should talk with their doctor before they start becoming more physically active
2. can start becoming more physically active by beginning slowly and building up gradually
3. has no signs or symptoms of coronary artery disease
4. should have an exercise test supervised by a physician before participating in moderate to vigorous intensity
exercise

Answer: 1
Cardiac, Vascular, and Pulmonary Systems
Examination
The Canadian Society of Exercise Physiology, developers of the PAR-Q, recommends anyone who answers “yes” to one of
more questions on the PAR-Q should talk with their doctor by phone or in person before starting to become much more
physically active. Only by answering “no” to all questions can the person be reasonably sure that they can start becoming
physically active and take part in a fitness appraisal. (American College of Sports Medicine p. 23)

5) A physical therapist prepares to apply a pneumatic intermittent compression device to a patient diagnosed with
venous insufficiency. When applying pneumatic intermittent compression to an extremity, the therapist should NOT
set the pressure of the unit above the:
1. patient’s diastolic blood pressure
2. patient’s systolic blood pressure
3. difference between the patient’s diastolic and systolic blood pressure
4. patient’s resting heart rate

Answer: 1
Non-Systems
Equipment & Devices: Therapeutic Modalities
A general guideline for the upper limit of pressure for pneumatic intermittent compression is the level of the patient’s
diastolic blood pressure. The value will ensure that the pressure does not approach the patient’s systolic value which could
result in a disruption of the patient’s arterial circulation. (Cameron p. 360)

6) A 65-year-old male status post CVA is relearning how to use his left upper extremity. The patient has minimal
movement of the arm and hand and demonstrates mild left neglect. He can complete gross flexor movements, but
has no active extension of the wrist and hand. When working with the patient, the therapist should:
1. sit on the right side of the patient’s body to assure that the patient is attending to the therapy session
2. avoid providing tactile cues to the patient’s involved hand and arm
3. encourage the patient to look at his left hand and arm while performing therapeutic activities
4. use the right upper extremity to compensate for the impaired left upper extremity

Answer: 3
Neuromuscular and Nervous Systems
Interventions
Strategies that direct the patient’s attention to the left side of his body and environment may be affecting in reducing the
patient’s neglect. Visual feedback may provide the patient with useful sensory information about his movement and use of
the involved arm and hand. (Carr p. 235)

7) A physical therapist is treating a 78-year-old female with advanced Alzheimer’s disease in the patient’s home. One
day, the physical therapist arrives at the patient’s home and finds that the patient’s daughter, who lives with the
patient, is in tears and expressing extreme frustration. The physical therapist should:

Scorebuilders 6
Practice Questions 3

1. contact the local agency on aging to report an abusive situation


2. recommend that the daughter consider respite care for her mom and provide her with information
3. begin the therapy session, focusing on the needs of the patient
4. try to be supportive to the daughter by saying “I know this is frustrating, but you really need to worry about your
mom right now”

Answer: 2
Non-Systems
Safety & Professional Roles: Teaching/Learning; Research
The physical therapist needs to recognize and acknowledge the stress placed on the caregivers of patients with Alzheimer’s
disease. In this case, there is no apparent abuse, however, the daughter is in need of assistance. Respite care may provide
the necessary care for the patient, while being supportive of the daughter’s needs. (Umphred p. 810)

8) A patient is observed using a toothbrush as an eating utensil. When asked, the patient is not able to name the
toothbrush and is also unable to describe the toothbrush’s use. This is an example of:
1. anomia
2. visual agnosia
3. impaired figure ground
4. cortical blindness

Answer: 2
Neuromuscular and Nervous Systems
Foundations for Evaluation, Differential Diagnosis, and Prognosis
With an anomia, the patient is unable to name the object, but can usually describe or demonstrate the use of the object. In
this case, the patient cannot do either; this is an example of a visual agnosia. (Umphred p. 843)

9) A physical therapist is working with a patient who has a long history of peripheral arterial occlusive disease in the
bilateral lower extremities. Last year the patient sustained a severe injury to the right lower extremity and has since
shown signs of chronic venous insufficiency. The patient is referred to physical therapy due to increasing edema in
the foot and lower leg. Given the patient’s past and current medical history, the physical therapist should select
which of the following strategies to effectively and safely reduce the patient’s edema?
1. active ankle pumps in supine
2. positioning in elevation
3. use of custom compression garments
4. heels rises in standing

Answer: 1
Cardiac, Vascular, and Pulmonary Systems
Interventions
Given the patient’s history of peripheral arterial occlusive disease, treatments such as compression and elevation are not
appropriate. Active ankle pumps with the patient in supine may be effective in reducing edema due to the associated
muscular contraction without reducing blood supply to the limb. (Kisner p. 712)

10) A patient is shown an x-ray that identifies the area of the spine impacted by a recently diagnosed herniated disk.
Assuming the arrow identifies the spinal segment involved, which of the following would be the MOST typical
finding during the examination?
1. imapaired triceps reflex
2. diminished strength of the wrist extensors
3. deminished sensation along the medial arm and forearm
4. paresthesias in the index, long and ring fingers.

Answer: 2
Neuromuscular and Nervous Systems
Foundations for Evaluation, Differential Diagnosis, and Prognosis
The arrow identifies the C6 vertebral level. In the cervical spine, each nerve root is named for the vertebra directly below
it. The most appropriate method to determine the vertebral level is to count them down from C1 (atlas). C1 is identifiable
by the presence of a small tubercle instead of a true spinous process. Characteristic findings with involvement at the C6
nerve root level include diminished sensation along the anterior arm, radial side of the hand to the thumb and the index

Scorebuilders 6
4 Practice Questions

finger; diminished strength in the biceps, supinator, and wrist extensors; and impaired reflexes of the biceps and
brachioradialis. (Magee p. 16)

11) A physical therapist prepares to order a wheelchair for a patient with complete C4 quadriplegia. Which type of
wheelchair would be the MOST appropriate for the patient?
1. hemichair with one arm drive
2. manual wheelchair
3. manual wheelchair with hand rim projections
4. power wheelchair

Answer: 4
Non-Systems
Equipment & Devices: Therapeutic Modalities
A patient with a spinal cord injury at the C4 level would not possess the requisite upper extremity musculature to propel a
manual wheelchair. A power wheelchair would be appropriate to meet the patient’s mobility needs. (Pierson p. 184)

12) A 55-year-old female with cervical pain has been referred to physical therapy. During the examination the therapist
concludes that the patient has 70 degrees of cervical spine rotation to both sides. The majority of cervical rotation
would occur:
1. at the atlanto-occipital joint
2. at the atlanto-axial joint
3. between C2 and C3
4. between C5 and C6

Answer: 2
Musculoskeletal System
Clinical Application of Foundational Sciences
About half of cervical rotation range of motion takes place at the atlanto-axial joint. (Hoppenfeld p. 114)

13) During an examination, a physical therapist determines that a patient’s respiratory rate is 30 breaths per minute.
What is the FIRST intervention the therapist should perform to help the patient slow his respiratory rate?
1. pursed-lip breathing
2. inspiratory hold
3. diaphragmatic breathing
4. active cycle of breathing

Answer: 1
Cardiac, Vascular, and Pulmonary Systems
Interventions
Pursed-lip breathing may be useful whenever an increase in breathing effort is noted. The intervention naturally slows
down respirations and decreases minute ventilation, relieving dyspnea in some patients. (Hillegass p. 736)

14) A physical therapist performs a series of lower extremity manual muscle tests on a patient with suspected unilateral
weakness. The therapist begins one of the tests by placing the patient in a prone position with the knee flexed to 90
degrees while extending the hip. The position is MOST appropriate for testing of the:
1. biceps femoris
2. gluteus medius
3. gluteus maximus
4. hamstrings

Answer: 3
Musculoskeletal System
Examination
In order to isolate the gluteus maximus during manual muscle testing, the hamstring muscles must be placed on slack at the
knee to remove their contribution. (Hislop p. 192)

15) A physical therapist observes a patient complete a test that requires the patient to perform quick alternating pronation
and supination of the forearm. This test can be used to identify:

Scorebuilders 6
Practice Questions 5

1. dysmetria
2. dysphagia
3. dysdiadochokinesia
4. dysarthria

Answer: 3
Neuromuscular and Nervous Systems
Examination
Dysdiadochokinesia is an impairment of rapid alternating movements that may be tested by performing quick alternating
forearm movements. (Umphred p. 724)

16) A 36-year-old female who gave birth to three children over the span of nine years is referred to physical therapy
during her fourth pregnancy. The patient presents with complaints of involuntary urine leakage when she coughs or
sneezes and when she participates in an aerobics class. Her physician tested for organ prolapse, however the test was
negative. What is the MOST appropriate term for the urinary leakage the patient is experiencing?
1. urge incontinence
2. overflow incontinence
3. functional incontinence
4. stress incontinence

Answer: 4
Other Systems
Foundations for Evaluation, Differential Diagnosis, and Prognosis
Stress incontinence is the involuntary loss of urine that occurs as a result of increases in intra-abdominal pressure. (Pauls p.
641)

17) A 62-year-old female with a transtibial amputation is examined in physical therapy. The patient has been using a
roll-on silicone liner inside of her prosthesis. While there are many benefits of this type of liner, one potential
disadvantage is:
1. increased shear force on the residual limb
2. reduced padding over bony prominences
3. more potential trauma to the suture line
4. heat buildup, especially in warmer climates

Answer: 4
Non-Systems
Equipment & Devices; Therapeutic Modalities
The silicone liner does not breathe as well as the fabric shrinker and therefore heat build up can be a problem. Advantages
to using the silicone liner include the roll-on design is usually less traumatic to the suture line than the fabric pull of the
shrinker (Seymour p. 129)

18) A physical therapist administers cardiopulmonary resuscitation (CPR) to a patient who collapsed in the waiting area.
The therapist is unable to establish a pulse after three cycles of CPR and therefore prepares to utilize an automated
external defibrillation unit. Which action would be the LEAST appropriate when using the device?
1. wipe the patient's chest dry
2. apply the pads to the patient's bare chest
3. analyze the patient's heart rhythm
4. ask another therapist to stabilize the patient's head when administering the electric shock

Non-Systems
Safety & Professional Roles; Teaching/Learning; Research
An automated external defibrillation (AED) unit can analyze a patient's heart rhythm and if necessary administer an
electrical shock to a patient in cardiac arrest. When using the AED the therapist should wipe the patient's chest dry and
apply the pads directly to the patient's chest. Assessing the patient's pulse by palpation does not provide specific
information on the patient's heart rhythm. Prior to administering an electrical current the therapist must make sure that no
one is in direct contact with the patient. (American Red Cross p. 60)

19) A physical therapist assistant employed in an acute care hospital works with a patient three days status post
abdominal surgery. While working bedside with the patient on lower extremity exercises in supine the patient

Scorebuilders 6
6 Practice Questions

suddenly complains of mild dizziness, fatigue, and nausea. The MOST IMMEDIATE physical therapist assistant
action is to:
1. take the patient's vital signs
2. assist the patient to a sitting position
3. contact the supervising physical therapist
4. call a code

Answer: 1
Other Systems
Interventions
The patient's symptoms are relatively common following surgery and therefore do not warrant external assistance at this
point. By taking the patient's vital signs the therapist can gather important objective data to further clarify whether the
patient would benefit from additional assistance. (Pierson p. 48)

20) A 25-year-old male basketball player is referred to outpatient physical therapy with lateral ankle instability. The
patient reports that he landed on another player's foot after coming down with a rebound and felt immediate pain. He
presents with moderate to severe ecchymosis and swelling along the lateral ankle distal to the lateral malleolus and is
tender to palpation in the same region. Which of the following methods would be the MOST appropriate to assess
the ligamentous stability of the ankle?
1. the therapist applies an anteriorly directed force with the patient in supine on the proximal tibia with the knee
flexed to 30 degrees
2. the therapist applies a compressive force to the heel with the patient in prone and rotates the tibia with the knee
flexed to 90 degrees
3. the therapist manually dorsiflexes the ankle with the patient in sitting with the knee in 90 degrees of flexion
4. the therapist stabilizes the tibia and fibula with the patient in supine with the foot in 20 degrees of plantar flexion
while drawing the talus forward in the ankle mortise

Answer: 4
Musculoskeletal System
Examination
The presented scenario describes a version of the anterior drawer test of the ankle which is designed to test the integrity of
the lateral ankle ligaments, most notably the anterior talofibular ligament. (Magee p. 801)

21) How does a pulmonary embolus affect the ventilation-perfusion relationship in the alveolar capillaries?
1. the ratio of ventilation to perfusion is normal; approximately 0.8
2. the ratio of ventilation to perfusion decreases to < 0.8
3. the ratio of ventilation to perfusion increases to > 0.8
4. the ratio of ventilation to perfusion is 0.0

Answer: 3
Cardiac, Vascular, and Pulmonary Systems
Foundations for Evaluation, Differential Diagnosis, and Prognosis
In a normal state of health, the lungs receive about 4 liters per minute of alveolar ventilation and about 5 liters of blood.
Thus the normal ventilation-perfusion ratio, or V/Q, is approximately 0.8. In the presence of a pulmonary embolus, blood
flow through the alveolar capillaries is decreased, while ventilation to the alveoli remains unchanged. Thus, decreasing the
denominator (Q) of the V/Q ratio, while the numerator (V) remains unchanged, causes the ratio to increase greater than 0.8.
This has the affect of increasing the ventilation to perfusion ratio. Normal ventilation with less than normal capillary
perfusion is called a physiologic dead space. (Brannon p. 58)

22) On radiographic imaging a patient has a sacral base that is angled 50 degrees anteriorly from the horizontal with the
patient in standing. What effect would this MOST likely have on the patient's standing posture?
1. the patient would exhibit normal posture
2. the patient would have an increased lumbar lordotic curve
3. the patient would have a decreased lumbar lordotic curve
4. the patient would have a posteriorly rotated pelvis

Answer: 2

Scorebuilders 6
Practice Questions 7

Musculoskeletal System
Clinical Application of Foundational Sciences
The normal sacral base is at approximately a 40 degree angle in relation to the horizontal. An increased sacral base angle
would tend to bring the lumbar spine into excessive lordosis. (Neumann p. 293)

23) A patient's respiratory exchange ratio is measured as 1:1 during a graded exercise test that includes an analysis of
expired gases. This indicates the patient is:
1. approaching his maximum aerobic capacity
2. no longer able to meet his metabolic need for O2 via anaerobic metabolism
3. able to meet his metabolic need for O2 via aerobic metabolism
4. able to meet his metabolic need for CO2 via anaerobic metabolism

Answer: 1
Cardiac, Vascular, and Pulmonary Systems
Examination
The respiratory exchange ratio (RER) is a ventilatory measurement that reflects gas exchange between the lungs and
pulmonary blood. The RER exceeds 1.0 during heavy, non-steady state or maximal exercise due to hyperventilation and
buffering of lactic acid in the blood. (American College of Sports Medicine p. 25)

24) A patient presents with an increase in lumbar lordosis and decreased flexibility in the rectus femoris. Based on this
information, the pelvis would tend to be positioned in a/an:
1. neutral pelvic tilt
2. posterior pelvic tilt
3. lateral pelvic tilt
4. anterior pelvic tilt

Answer: 4
Musculoskeletal System
Foundations for Evaluation, Differential Diagnosis, and Prognosis
Based on the origin and insertion of the rectus femoris, a decrease in the muscle's normal resting length would pull the
anterior superior iliac spine of the ilium anteriorly thereby moving the pelvis into an anterior pelvic tilt. (Levangie p. 419)

25) A physical therapist works with a female patient who is in her third trimester of pregnancy. The patient complains of
persistent low back pain, radiating from the sacrum to the hip joint along the sciatic nerve distribution. The therapist
determines that tightness in the right piriformis muscle is contributing to the patient's discomfort. In order to stretch
the muscle, the patient should be positioned on her left side with the affected hip in:
1. medial rotation and adduction
2. lateral rotation and adduction
3. medial rotation and abduction
4. lateral rotation and abduction

Answer: 1
Other Systems
Interventions
The piriformis muscle assists in abducting and laterally rotating the thigh. As a result, the patient needs to move the
involved lower extremity toward hip medial rotation and adduction to stretch the muscle. This is a modified position from
the standard stretch in supine or quadruped secondary to the patient's pregnancy. Tightness or spasm of the piriformis
muscle can irritate the sciatic nerve, causing pain in the buttocks and referred pain along the course of the sciatic nerve.
This referred pain, called "sciatica", often progresses down the back of the thigh and/or into the low back. (Stephenson p.
194)

26) A physical therapist uses electromyographic biofeedback to improve a patient's ability to recruit the vastus medialis
obliquus muscle. After initiating the intervention the therapist observes that the patient is unable to generate the
anticipated level of audio and visual activity on the biofeedback unit. The MOST appropriate therapist action is
1. modify the location of the surface electrodes
2. increase the size of the surface electrodes

Scorebuilders 6
8 Practice Questions

3. remove oil and dead skin from the target area using an alcohol prep pad
4. select a more conservative strengthening intervention

Answer: 1
Musculoskeletal System
Interventions
When using electromyographic biofeedback the electrodes must be placed as close to the muscle being monitored as
possible. Failure to achieve appropriate electrode placement can result in inadequate audio or visual activity or an increase
in extraneous electrical activity. If the surface electrodes are determined to be in an appropriate location it may be necessary
to increase the sensitivity setting on the biofeedback unit. (Prentice-Therapeutic Modalities p. 190)

27) A patient with multiple sclerosis is examined in physical therapy. The physical therapist determines that the patient's
right shoulder is noticeably higher than the left due to the presence of a spinal deformity. The therapist notes that the
patient has difficulty keeping his feet on the foot rests of his wheelchair and has a bright red area of skin over the
right ischial tuberosity. The MOST likely cause of the patient's skin redness is:
1. increased muscle tone on the right side
2. inadequate distribution of weight bearing forces
3. failure to do weight shifting exercises on a regular basis
4. sensory impairment

Answer: 2
Other Systems
Interventions
All of the listed factors can potentially contribute to causing skin redness, however, an appropriate seating system that
effectively distributes weight bearing forces will minimize any risk of skin breakdown in spite of the remaining factors.
(O'Sullivan p. 1066)

28) A physical therapist reviews an examination for a patient with a C3 spinal cord injury. Which description would be
MOST consistent with the patient's level of injury?
1. minimal assistance for bathing activities
2. minimal assistance to modified independent for sliding board transfers
3. independent with assistive devices for verbal communication
4. modified independent for feeding with adaptive equipment

Answer: 3
Neuromuscular and Nervous Systems
Foundations for Evaluation, Differential Diagnosis, and Prognosis
An individual with a complete C3 spinal cord injury will require the use of adaptive devices to communicate since the
diaphragm and muscles of inspiration are paralyzed. Individuals with C1-C3 involvement require artificial ventilation due
to the loss of phrenic nerve innervation. All other options require a lower level of injury where certain upper extremity
muscle groups have been preserved. (Umphred p. 491)

29) A physical therapist employed by a home health agency receives a new referral for a 27-year-old male with a
traumatic brain injury who was recently discharged from a rehabilitation hospital. During the examination the patient
becomes combative and appears to be extremely agitated. The therapist and the patient's spouse attempt to calm the
patient for approximately ten minutes without any success. The MOST IMMEDIATE therapist action is:
1. continue with the examination as planned
2. attempt to condense the remaining portion of the examination
3. document the patient's behavior in the medical record
4. discontinue the treatment session

Answer: 4
Neuromuscular and Nervous Systems
Interventions
The patient's present state makes it unreasonable to continue with the examination and as a result the therapist should
discontinue the treatment session. Failure to act in this manner could result in an unnecessary safety risk for all involved

Scorebuilders 6
Practice Questions 9

parties. Documenting the patient's behavior is an appropriate option, however it would not be the most immediate therapist
action. (O'Sullivan p. 791)

30) A physical therapist administers grade I and II mobilizations to the left shoulder of a patient after completing a series
of upper extremity resistive exercises. The PRIMARY purpose of the intervention is to:
1. increase periarticular extensibility
2. decrease pain
3. reduce hypomobility
4. facilitate muscle recruitment

Answer: 2
Musculoskeletal System
Interventions
Grade I or II oscillations or slow intermittent grade I or II sustained joint distractions are primarily utilized to decrease pain.
(Kisner p. 224)

31) A physical therapist examines a patient with a venous ulcer on the medial side of her lower leg. The therapist
observes that the ulcer penetrates through the subcutaneous tissue partially exposing several tendons. The MOST
appropriate grade for the ulcer using Wagner's Ulcer Grade Classification Scale is:
1. grade 1
2. grade 2
3. grade 3
4. grade 4

Answer: 2
Integumentary System
Examination
The Wagner Ulcer Grade Classification system is used to establish the presence of depth and infection in a wound. The
scale is used for arterial, venous or neuropathic ulcers and is not appropriate for pressure ulcers. The system uses a 0-5
ordinal grading scale. A grade of 0 represents preulcerative lesions; healed ulcers; or presence of bony deformity, while a
grade of 5 represents gangrene of the foot requiring disarticulation. (Goodman-Pathology p. 308)

32) A therapist examines a 58-year-old patient who is recovering from a humeral neck fracture. The therapist determines
that all passive and active movements of the shoulder joint are moderately restricted. Which of the following
techniques would be the MOST appropriate to improve the patient's lateral rotation?
1. anterior gliding of the head of the humerus on the glenoid fossa
2. posterior gliding of the head of the humerus on the glenoid fossa
3. inferior gliding of the head of the humerus on the glenoid fossa
4. superior gliding of the head of the humerus on the glenoid fossa

Answer: 1
Musculoskeletal System
Interventions
The glenohumeral joint consists of a concave glenoid fossa and a convex humeral head. Anterior gliding will tend to stretch
the anterior capsule which will allow for an increase in lateral rotation. The mobilization should occur with the patient in
prone with the arm in a relaxed position over the edge of a treatment plinth. (Kisner p. 232)

33) A patient three days status post myocardial infarction is medically ready to begin a formalized, inpatient exercise
program. The MOST appropriate guideline to use when determining the patient's upper limit to exercise is:
1. resting respiratory exchange ratio
2. respiratory quotient
3. maximum oxygen consumption
4. resting heart rate plus 20 beats per minute

Answer: 4

Scorebuilders 6
10 Practice Questions

Cardiac, Vascular, and Pulmonary Systems


Interventions
Recommendations for inpatient exercise programming for a patient status post myocardial infarction include a heart rate
less than 120 beats per minute or a resting heart rate plus 20 beats per minute. (American College of Sports Medicine p.
169)

34) A physical therapist discusses positioning with a 42-year-old female patient rehabilitating from a transfemoral
amputation. Based on the patient's postoperative status, which type of contracture is the patient MOST susceptible
to?
1. hip flexor and hip extensor
2. hip extensor and hip adductor
3. hip flexor and hip abductor
4. hip extensor and hip abductor

Answer: 3
Musculoskeletal System
Interventions
The relative strength of the hip flexors and abductors compared to the hip extensors and adductors contribute to an
increased incidence of flexion and abduction contractures. The presence of a contracture can significantly impair a patient's
ability to ambulate with a prosthesis. (Seymour p. 145)

35) A 21-year-old male with cerebral palsy uses a wheelchair for basic mobility. The patient has significant difficulty
maintaining head and trunk control and has developed scoliosis and a resultant pelvic obliquity. What other
deformity was likely accommodated for based on the pictured adaptive seating design?

1. sacral sitting
2. decorticate rigidity
3. thoracic kyphosis
4. windswept hips

Answer: 4
Other Systems
Foundations for Evaluation, Differential Diagnosis, and Prognosis
The seat and foot platform are shifted to the right of midline indicating a windswept or windblown deformity. Also note the
abductor pommel and large lateral thigh wedges. This occurs when both knees go to one side; one hip is flexed, adducted,
and internally rotated with flexion and abduction of the opposite hip. (Bergen p. 376)

36) A physical therapist treats a patient status post open reduction internal fixation secondary to a hip fracture. While
preparing to transfer the patient the therapist notes that the calf on the surgical side is swollen and extremely warm to
touch. The patient complains of pain in the calf with passive dorsiflexion and reports mild discomfort to light touch.
The MOST likely explanation is:
1. compartment syndrome

Scorebuilders 6
Practice Questions 11

2. deep vein thrombosis


3. infection
4. peroneal nerve palsy

Answer: 2
Other Systems
Interventions
Deep vein thrombosis may occur secondary to a period of immobility. The calf is an extremely common site for a thrombus
to develop which is characterized by swelling, discomfort, increased warmth to touch, and a positive Homans' sign. (Paz p.
400)

37) A physical therapist assistant treats a patient rehabilitating from knee surgery. The supervising physical therapist had
previously discussed the patient's established treatment plan and each associated activity with the physical therapist
assistant. While exercising on the stationary bicycle the physical therapist assistant leaves the patient unattended.
While trying to get off the bicycle the patient falls and fractures their hip. Which of the following individuals would
be MOST liable for the incident?
1. supervising physical therapist
2. physical therapist assistant
3. referring physician
4. patient

Answer: 2
Non-Systems
Safety & Professional Roles; Teaching/Learning; Research
The scenario describes an appropriate delegation from a physical therapist to a physical therapist assistant. By failing to
monitor the patient during the exercise activity the physical therapist assistant violated the established standard of care and
therefore placed the patient at unnecessary risk. The physical therapist would likely share in the liability based on the
actions of the physical therapist assistant, however, the most liable individual would remain the physical therapist assistant.
Physical therapist assistants are licensed health care providers in the vast majority of states. (Scott - Professional
Malpractice p. 21)

38) A patient in an acute care hospital complains to his physical therapist about point tenderness in his back following a
diagnostic test earlier in the day. An entry in the medical record reveals that the patient had a lumbar puncture. Based
on the given diagnostic test, at what spinal level would the therapist expect the patient to be the MOST tender?
1. T12-L1
2. L1-L2
3. L2-L3
4. L3-L4

Answer: 4
Other Systems
Foundations for Evaluation, Differential Diagnosis, and Prognosis
A lumbar puncture is a diagnostic test that allows for sampling of the cerebrospinal fluid for analysis. The test can be used
to confirm the presence of a number of serious medical conditions including viral or bacterial meningitis. The test is
performed by inserting a needle in the midline of the spine at the L3-L4 or L4-L5 level. (Nurses 3 Minute Clinical
Reference p. 907)

39) A physical therapist works with a physical therapist assistant on a rehabilitation floor in an acute care hospital. After
a busy morning the physical therapist assistant reports to the physical therapist about the activities they performed.
Which activity would be the LEAST appropriate for the physical therapist assistant?
1. writing a discharge summary in the medical record
2. progressing a patient within an established plan of care
3. reporting at a team conference
4. conducting a formal test or measurement

Answer: 1

Scorebuilders 6
12 Practice Questions

Non-Systems
Safety & Professional Roles; Teaching/Learning; Research
Physical therapist assistants often complete documentation in the medical record, however the physical therapist is
responsible for establishing a discharge plan and documenting the discharge summary. Physical therapist assistants can
progress patients as long as the progression is within the boundaries of the established plan of care. (Guide to Physical
Therapist Practice p. 42)

40) A 78-year-old female falls to the ground while ambulating in an assisted living facility. The patient is found lying
supine with her right hip adducted and medially rotated. Upon examination, the greater trochanter is particularly
prominent on the right side and the right limb appears shorter than the left. The patient reports severe right hip pain
when she attempts to move the right lower extremity. The clinical presentation is MOST indicative of:
1. traumatic femur fracture
2. traumatic posterior hip dislocation
3. traumatic intertrochanteric fracture
4. traumatic anterior hip dislocation

Answer: 2
Musculoskeletal System
Foundations for Evaluation, Differential Diagnosis, and Prognosis
Patients with a posterior hip dislocation often present with the involved limb in adduction and medial rotation. The greater
trochanter may be particularly prominent and the involved limb often appears to be shorter than the contralateral limb.
(Brotzman p. 443)

41) A young, healthy adult is breathing at tidal volume. What percentage of the tidal volume reaches the respiratory units
and is available for gas exchange?
1. 70%
2. 80%
3. 90%
4. 100%

Answer: 1
Cardiac, Vascular, and Pulmonary Systems
Clinical Application of Foundational Sciences
The tidal volume of a young, healthy adult is approximately 500 mL. Of the 500 mL that enters the respiratory system, the
volume of air that reaches the respiratory units and takes part in gas exchange is about 350 mL. The remaining 150 mL of
air occupies the conducting airways and is considered to be within the anatomic dead space because it does not take part in
gas exchange. (Hillegass p. 55)

42) A physical therapist performs a series of exercises with a patient in a therapeutic pool to help improve their general
strength. The therapist wants to challenge the patient and increase resistance during exercise. The BEST way to
increase resistance is to move the extremity:
1. rapidly upward toward the surface of the water
2. slowly upward toward the surface of the water
3. slowly downward away from the surface of the water
4. rapidly downward away from the surface of the water

Answer: 4
Musculoskeletal System
Interventions
The greatest resistance to movement in the water is created by rapid motion in the direction away from the surface. Based
on the properties of water, resistance to movement has a linear relationship with the velocity of movement. When moving
upward toward the surface, buoyancy will assist the motion. When moving downward away from the surface, resistance
increases due to the increase in the hydrostatic pressure of water. (Cameron p. 263)

Scorebuilders 6
Practice Questions 13

43) A patient successfully completes six weeks of aerobic exercise conditioning in a supervised outpatient cardiac
rehabilitation program. On the last day of rehabilitation, while exercising on a bicycle ergometer, the patient
classifies the workload as a 13 using a rate of perceived exertion scale. The workload should be:
1. the same workload that elicited a rating of 13 on the first day of cardiac rehabilitation
2. a lower workload than what elicited a rating of 13 on the first day of cardiac rehabilitation
3. a higher workload than what elicited a rating of 13 on the first day of cardiac rehabilitation
4. the same workload that elicited a rating of 6 on the first day of cardiac rehabilitation

Answer: 3
Cardiac, Vascular, and Pulmonary Systems
Interventions
The expected outcome of successful aerobic conditioning is improved exercise tolerance. Physiologic benefits of regular
physical activity include increased maximal oxygen consumption and lower heart rate and blood pressure for a given
absolute submaximal intensity. A patient with improved exercise tolerance would be able to exercise at a higher workload
before reaching the equivalent rate of perceived exertion (RPE). (American College of Sports Medicine p. 5)

44) A physical therapist completes a home assessment for a 39-year-old male who uses a wheelchair for household and
community mobility. In order for the patient to enter his home, the doorway width should be a MINIMUM of:
1. 24 inches
2. 28 inches
3. 32 inches
4. 36 inches

Answer: 3
Non-Systems
Safety & Professional Roles; Teaching/Learning; Research
The Americans with Disabilities Act indicates that 32 inches is the minimum recommended width to accommodate most
wheelchairs. (Minor p. 472)

45) A patient with complete C6 quadriplegia is learning to perform a wheelchair to mat transfer. Which of the following
muscles can effectively lock the patient's elbows during the transfer?
1. anterior deltoid
2. biceps
3. triceps
4. latissimus dorsi

Answer: 1
Other Systems
Interventions
Since the triceps are not innervated in a patient who has a complete lesion at the C6 level, the patient must substitute
another muscle to perform this task. When the patient's hand is in a weight bearing position on the mat, the anterior deltoid
can be used to lock the elbow. (O'Sullivan p. 898)

46) A physical therapist designs a research project to examine the efficacy of biofeedback as a treatment for urinary
stress incontinence. In preparation for the study, the physical therapist drafts an informed consent document as part
of the process for approval to initiate the project at his clinical facility. Which of the following statements would
violate the validity of the informed consent document?
1. There is no compensation available from the facility should an injury occur
2. Participants must complete all the sessions outlined in the study in order to receive physical therapy services for
their condition.
3. Participants must contact their own primary care physician in case of injury.
4. Participants may not begin any other new treatment program for the duration of the study.

Answer: 2
Non-Systems

Scorebuilders 6
14 Practice Questions

Safety & Professional Roles; Teaching/Learning; Research


The informed consent document must include a statement that the subject is free to withdraw consent at any time without
penalty. (Domholdt p. 420)

47) A physical therapist treats a patient rehabilitating from a serious hand and wrist injury sustained in a work related
accident three months ago. The patient has made good progress in physical therapy, but still has severe difficulty
with activities requiring opposition of the thumb. Which type of grip should be the easiest for the patient to perform?
1. digital prehension
2. three point chuck
3. lateral prehension
4. tip pinch

Answer: 3
Musculoskeletal System
Clinical Application of Foundational Sciences
Lateral prehension is the only listed grip that does not require opposition of the thumb and fingers. Lateral prehension refers
to a grip where the thumb and lateral side of the index finger come into contact. An example would be holding a key or a
hand of playing cards. (Magee p. 379)

48) A physical therapist inspects the palmar surface of the hand of a 57-year-old male recently referred to physical
therapy. During the inspection the therapist palpates several nodules in the palmar aponeurosis immediately inferior
to the ring and little finger. This type of finding is MOST commonly associated with:
1. boutonniere deformity
2. Dupuytren's contracture
3. de Quervain's tenosynovitis
4. Bouchard's nodes

Answer: 2
Musculoskeletal System
Clinical Application of Foundational Sciences
Dupuytren's contracture is characterized by a progressive painless thickening and tightening of the palmar fascia resulting
in a fixed flexion deformity of the metacarpophalangeal and proximal interphalangeal joints most often in the ring or little
finger. Typically with this condition the skin becomes adherent to the fascia with the presence of nodules. (Magee p. 366)

49) Physical therapists use heart rate as a measure of the intensity of aerobic exercise because heart rate has a:
1. negative linear relationship with age
2. positive linear relationship with age
3. positive linear relationship with oxygen consumption
4. negative linear relationship with oxygen consumption

Answer: 3
Non-Systems
Safety & Professional Roles; Teaching/Learning; Research
Heart rate is used as a guide to set exercise intensity because of the relatively linear relationship between heart rate and
oxygen consumption. (American College of Sports Medicine p. 143)

50) A physical therapist examines a patient two weeks status post right total knee arthroplasty. During the examination
the therapist notes a significant restriction of passive and active inferior patellar mobility. Which of the following
would be the MOST likely clinical finding?
1. diminished active and passive knee flexion
2. presence of an extension lag
3. increased pain with active knee range of motion
4. diminished knee flexion strength

Answer: 1

Scorebuilders 6
Practice Questions 15

Musculoskeletal System
Foundations for Evaluation, Differential Diagnosis, and Prognosis
A restriction of inferior patellar mobility often coincides with a loss of knee flexion since the patella is required to track
inferiorly. (Kisner p. 249)

51) A physical therapist notices that a 52-year-old male diagnosed with Parkinson's disease frequently experiences
?freezing episodes.? The therapist hypothesizes that the episodes may be related to several recent changes in the
patient's medications. Which physical therapy management technique would be the MOST appropriate during the
described episodes?
1. passive range of motion exercises and positioning instructions
2. strengthening exercises for the trunk and all four extremities
3. gait training activities with appropriate bracing
4. electrical stimulation to the ?non-functioning? muscles

Answer: 1
Neuromuscular and Nervous Systems
Interventions
During a ?freezing episode? the levodopa or combination of medications may not be working optimally, allowing for
increased stiffness and a decrease in the ability to actively move. Therefore, the patient should be seen for more passive
interventions such as range of motion and positioning until he can begin moving more freely again. (Goodman-Pathology p.
1047)

52) A physical therapist prepares to transfer a 62-year-old male patient from supine to short sitting. The patient was
admitted to a skilled nursing facility two weeks ago and currently requires maximum assistance to complete all
transfers. When transferring the patient it is MOST critical to:
1. provide clear and concise verbal commands
2. utilize appropriate guarding technique
3. lead with the patient's stronger side
4. maintain direct contact with the patient

Answer: 2
Non-Systems
Safety & Professional Roles; Teaching/Learning; Research
The patient's sitting balance is likely to be compromised since the patient requires maximum assistance to complete all
transfers. By utilizing appropriate guarding technique the therapist has the best opportunity to address the myriad of
potential problems that could arise during the transfer. (Pierson p. 135)

53) A 20-year-old female is having difficulty learning how to perform diaphragmatic breathing. To facilitate the patient's
learning the physical therapist should:
1. have her exhale through pursed lips as if to make a candle flame flicker
2. have her use a flutter valve
3. have her sniff
4. place her in the Trendelenburg position

Answer: 3
Cardiac, Vascular, and Pulmonary Systems
Interventions
Sniffing is a training technique that naturally enlists the diaphragm. The therapist should instruct the patient to sniff quickly
through the nose three times followed by slow relaxed exhalations. The therapist should assess whether the patient is
showing a diaphragmatic breathing pattern. (Hillegass p. 660)

54) A physical therapist observes a patient with a transtibial amputation who is donning his prosthesis. After the patient
completes this task you notice that his residual limb is not fully in the socket and he complains of pain on the tibial
tubercle. To resolve this problem the physical therapist should:
1. call the prosthetist and ask him to alter the socket

Scorebuilders 6
16 Practice Questions

2. remove a one-ply residual limb sock


3. add a one-ply residual limb sock
4. initiate a desensitization program

Answer: 2
Musculoskeletal System
Interventions
The patient's limb is not fully in the socket and the pain on the tibial tubercle is likely due to its contact with the patellar-
tendon bearing bar. Thus, it appears as if the patient has applied too many residual limb socks. To remedy this, one sock
should be removed at a time until the residual limb fits appropriately within the socket. (Seymour p. 190)

55) A physical therapist is concerned about the potential for a patient to develop skin breakdown over the sacral and
ischial regions. As a result the therapist closely inspects the patient's skin twice daily during physical therapy
treatment. Which of the following is the earliest sign of a pressure ulcer that can be obtained through inspection?
1. macerated tissue
2. localized redness
3. pale or blanched skin
4. decubitus ulceration

Answer: 2
Integumentary System
Examination
Stage 1 of a pressure ulcer is characterized by local redness that does not blanch when pressure is applied to the area. The
epidermis and dermis layers of the skin remain intact. (Pierson p. 302)

56) A physical therapist examines a 29-year-old female referred to physical therapy with low back pain. The patient is 22
weeks pregnant and according to the medical record she frequently experiences supine hypotension. Based on the
patient's medical condition the MOST appropriate position to avoid the effects of supine hypotension would be:
1. supine with both knees bent
2. right sidelying
3. left sidelying
4. supine with head elevated on two pillows

Answer: 3
Other Systems
Interventions
After the fourth month of pregnancy the vena cava can be compressed by the uterus when positioned in supine. The
decrease in venous return and subsequent decrease in cardiac output can result in supine hypotension syndrome. Positioning
a patient in left sidelying will place the smallest amount of pressure on the aorta and therefore allows for the greatest
increase in cardiac output. (Kisner p. 684)

57) A physical therapist performs grip strength measurements using a handheld dynamometer on a patient rehabilitating
from a work related accident. The patient has been in physical therapy for four weeks and should be able to return to
work in approximately two additional weeks. After the treatment session the therapist records in the patient's chart
that grip strength varied by approximately 25% in a test-retest situation. The MOST plausible hypothesis is:
1. the patient is susceptible to fatigue
2. the patient is not exerting maximal effort
3. the patient experienced an increase in pain with repeated muscle contraction
4. the patient sustained an exacerbation of the original injury

Answer: 2
Musculoskeletal System
Foundations for Evaluation, Differential Diagnosis, and Prognosis
A relatively large discrepancy in force production (over 20%) in a test-retest situation is often an indication that the patient
is not exerting maximal effort. On occasion patients can attempt to manipulate this form of objective measure in order to
secure or reinforce some type of secondary gain. (Magee p. 380)

Scorebuilders 6
Practice Questions 17

58) A physical therapist works with a 36-year-old male diagnosed with rotator cuff tendonitis. During the session the
therapist asks the patient to medially rotate the arm behind the back with the dorsum of the hand resting in the mid-
lumbar region. The therapist then instructs the patient to attempt to lift the hand off the back. This test is MOST
appropriate to assess the:
1. infraspinatus
2. subscapularis
3. supraspinatus
4. teres minor

Answer: 2
Musculoskeletal System
Examination
Attempting to lift the hand off the back requires the patient to medially rotate the arm. The subscapularis serves as a medial
rotator of the shoulder when the arm is positioned by the side. The muscle is innervated by the subscapular nerve (C5-C6).
(Magee p. 279)

59) A physical therapist working in an outpatient physical therapy clinic attempts to determine the appropriateness of
delegating a specific patient care activity to a physical therapist assistant. Which factor would be the MOST critical
to assess when making this type of decision?
1. the physical therapist has previously instructed the patient in the activity
2. the activity is within the scope of practice of the physical therapist assistant
3. the physical therapist will be in the same treatment area in the event the physical therapist assistant needs
assistance
4. the patient expresses a willingness to work with the physical therapist assistant

Answer: 2
Non-Systems
Safety & Professional Roles; Teaching/Learning; Research
Physical therapists should only delegate patient care activities to physical therapist assistants that are within their
established scope of practice. Physical therapist assistants are licensed in the majority of states and therefore do not require
direct supervision at all times nor do physical therapists necessarily have to instruct a patient in a given activity prior to the
patient working with the physical therapist assistant. (Guide to Physical Therapist Practice p. 42)

60) A physical therapist observes an infant positioned in prone. Which component of the developmental sequence is the
infant BEST exhibiting?

1. pivot prone
2. prone on elbows
3. modified plantigrade
4. hooklying rotation

Answer: 1
Neuromuscular and Nervous Systems

Scorebuilders 6
18 Practice Questions

Clinical Application of Foundational Sciences


Pivot prone is exhibited when the infant is in prone with extension of the neck, spine, and hips as well as adduction of the
scapulae with lateral rotation of the shoulders. (Tecklin p. 13)

61) A physical therapist treats a patient using pulsed ultrasound at 3 MHz. Which patient scenario would MOST warrant
the use of this intervention as part of the plan of care?
1. a 32-year-old male with decreased knee range of motion secondary to tight quadriceps muscles following cast
removal
2. a 24-year-old female with patellar tendonitis
3. a 22-year-old male with a chronic hamstrings muscle strain
4. a 42-year-old female with decreased ankle complex range of motion secondary to a tight gastrocnemius/soleus
complex

Answer: 2
Non-Systems
Equipment & Devices; Therapeutic Modalities
Pulsed ultrasound is used when little or no heating is desired. Pulsed ultrasound is useful in acute conditions to decrease
pain and promote healing. 3 MHz ultrasound specifically targets superficial structures and is often used in areas of little
muscle bulk. (Cameron p. 196)

62) A physical therapist working in a rehabilitation hospital treats a patient with T10 paraplegia. The patient was injured
approximately six weeks ago in a snowmobile accident and was admitted to the rehabilitation hospital after a 28 day
stay in an acute care hospital. Assuming an unremarkable recovery, at the conclusion of rehabilitation the patient
would MOST likely:
1. require daily assistance from a home health aide for activities of daily living
2. utilize a wheelchair for functional mobility
3. utilize orthotics and a walker for household and community ambulation
4. be unable to safely return to his home

Answer: 2
Neuromuscular and Nervous Systems
Foundations for Evaluation, Differential Diagnosis, and Prognosis
A patient with T10 paraplegia would require a wheelchair for functional mobility due to the lack of active motion in the
lower extremities combined with the high energy cost of ambulation with bracing and an assistive device. (O'Sullivan p.
899)

63) A physical therapist explains the potential benefits of pursed-lip breathing to a physical therapist assistant with
limited experience using breathing exercises. The physical therapist explains that when a patient is performing
pursed-lip breathing correctly, the therapist should identify:
1. a decrease in the patient's arterial oxygen saturation
2. an increase in the patient's arterial carbon dioxide saturation
3. adventitious breath sounds during lung auscultation
4. a longer expiratory phase than inspiratory phase

Answer: 4
Cardiac, Vascular, and Pulmonary Systems
Interventions
Instructions for pursed-lip breathing are to breathe in through the nose with your lips firmly together except at the very
center. Attempt to breathe out twice as long as you breathe in. The benefits of pursed-lip breathing are decreased frequency
of respiration, increased tidal volume, and improved control of breathing during normal activities and periods of distress.
(American College of Sports Medicine p. 202)

64) A physical therapist performs an examination on a patient with a pressure ulcer on the heel. The patient's wound is
relatively round, approximately one half centimeter in size, and is covered with black necrotic tissue. In order to
determine the stage of the pressure ulcer the physical therapist must:

Scorebuilders 6
Practice Questions 19

1. estimate the depth of the ulcer based on the thickness of tissue typically present on the heel
2. make precise measurements of the surface area (i.e. perimeter) of the wound
3. first debride the wound of the necrotic tissue
4. gather information pertaining to the cause of the wound

Answer: 3
Integumentary System
Examination
Pressure ulcers are staged based on wound depth. In this case, the wound is covered by necrotic tissue which must be
removed via debridement before the ulcer's stage can be determined. (Kloth p. 174)

65) A physical therapist observes a five-month-old infant interact in a play session with her mother. Assuming normal
development, which activity would the therapist LEAST likely observe?
1. holds toy with two hands
2. pull to sit with head in alignment
3. forward propped position with anterior/posterior movement
4. quadruped position with rocking

Answer: 4
Neuromuscular and Nervous Systems
Clinical Application of Foundational Sciences
Typical motor acquisition during the fifth month includes gaining control of fundamental postures through sitting with
support, attaining a ?hands and knees? position, standing with support, grasping and releasing toys, and turning the head
while in sitting. An infant will not normally demonstrate rocking behavior in quadruped until the seventh through ninth
months. The rocking provides sensory feedback to the vestibular system and the extremities in this position. (Tecklin p. 17)

66) A physical therapist performs ambulation activities with a 66-year-old female status post total hip replacement. The
patient has an intravenous line (IV) inserted in the dorsum of her right hand. As the patient begins to ambulate the
therapist hears an alarm coming from the intravenous machine. The MOST IMMEDIATE therapist action is:
1. call for a nurse since this may represent a medical emergency
2. alert the nurse that the intravenous line has infiltrated
3. turn off the machine until you return the patient to her room
4. check for an obstruction to flow

Answer: 4
Non-Systems
Safety & Professional Roles; Teaching/Learning; Research
Physical therapists are not permitted to adjust, alter, modify or otherwise correct the IV system. Physical therapists can
perform simple procedures such as straightening the tubing or removing an object that is occluding the tubing. (Pierson p.
267)

67) A physical therapist receives a referral for a 48-year-old male with shoulder pathology. The physical therapy
examination indicates that the drop arm test was positive, however magnetic resonance imaging had earlier
confirmed that the rotator cuff was not torn. Which condition would be MOST likely to produce the objective
finding?
1. axillary nerve palsy
2. multidirectional instability
3. thoracic outlet syndrome
4. Bell's palsy

Answer: 1
Other Systems
Foundations for Evaluation, Differential Diagnosis, and Prognosis
A drop arm test is typically associated with a large rotator cuff tear, however, axillary nerve palsy is a condition that could
also limit the patient's ability to control their arm as it suddenly drops to the side. The axillary nerve (C5-C6) innervates the
deltoid muscle which functions as a primary abductor of the shoulder. (Reider p. 50)

Scorebuilders 6
20 Practice Questions

68) Which stimulus causes a decrease in heart rate and force of contraction?
1. neural stimulation by the vagus nerve
2. neural stimulation by the sympathetic center in the medulla oblongata
3. increased levels of carbon dioxide in the blood
4. decrease in blood Ph

Answer: 1
Other Systems
Clinical Application of Foundational Sciences
The parasympathetic division of the autonomic nervous system innervates the heart via the vagus nerve. Simulation of the
vagus nerve causes a release of acetylcholine which slows the intrinsic rate of the heart and decreases the force of
contraction. Vagal stimulation also enhances coronary blood flow by dilating the coronary arteries. Neural stimulation by
the sympathetic division of the autonomic nervous system has the opposite effect. Stimulation of the sympathetic center in
the medulla oblongata increases heart rate and force of contraction primarily through the Beta1 receptors in the atria, SA
node, AV node, and ventricles. Increased levels of carbon dioxide in the blood and decrease in blood pH have the same
inhibitory effect on heart rate and force of contraction. Their effects are mediated by the chemoreceptors in the carotid
body. (Hillegass p. 49)

69) A physical therapist administers a survey at a community health fair to identify individuals at risk for osteoporosis.
Which of the following items would be MOST closely associated with osteoporosis?
1. early menopause
2. obesity
3. African American race
4. high bone mineral density

Answer: 1
Other Systems
Foundations for Evaluation, Differential Diagnosis, and Prognosis
Menopause refers to the stage at which the ovaries no longer perform the functions of ovulation and estrogen production,
which results in accelerated bone loss. Early menopause is generally considered to occur prior to 40 years of age and places
the patient at significant risk for osteoporosis. (Edelman p. 302)

70) A 70-year-old male status post right hip fracture with open reduction internal fixation is referred to physical therapy
for gait training. The physician has ordered the patient to remain non-weight bearing on the right lower extremity for
four weeks. What is the MOST appropriate gait training method?
1. ambulation with bilateral crutches and a four-point gait pattern
2. ambulation with a walker and a three-point gait pattern
3. ambulation with a single point cane and a three-point gait pattern
4. ambulation with a quad cane and a three-point gait pattern

Answer: 2
Non-Systems
Equipment & Devices; Therapeutic Modalities
Ambulation with a walker using a three-point gait pattern is recommended when the patient is able to bear weight on one
extremity, but is non-weight bearing on the other. A cane or quad cane is not an appropriate assistive device for a patient
that is non-weight bearing. (Pierson p. 233)

71) A physical therapist works with a 27-year-old male status post open reduction and internal fixation of a tibia fracture.
The patient was referred to physical therapy for three sessions and is currently attending his final session prior to
discharge from the hospital. Assuming the patient has been cleared for partial weight bearing, the MOST appropriate
assistive device for the patient to use upon discharge is:
1. bilateral canes
2. crutches

Scorebuilders 6
Practice Questions 21

3. cane
4. walker

Answer: 2
Non-Systems
Equipment & Devices; Therapeutic Modalities
Crutches would accommodate the patient's current weight bearing status and provide the patient with the necessary support.
A cane or bilateral canes do not permit partial weight bearing and instead are primarily used to promote balance. A walker
would be acceptable, however, may limit the patient's mobility compared to crutches. (O'Sullivan p. 425)

72) A physical therapist attempts to assess proprioception in a patient rehabilitating from a prolonged hospitalization.
Which method would provide the therapist with the desired information?
1. the therapist moves the patient's upper extremity and asks the patient to indicate the direction of movement while
the extremity is in motion
2. the patient moves the upper extremity and indicates the direction of movement while the extremity is in motion
3. the therapist moves the patient's upper extremity to a selected position and asks the patient to describe the position
4. the patient moves the upper extremity to a selected position and is asked to describe the position

Answer: 3
Neuromuscular and Nervous Systems
Examination
Proprioception measures the ability of a patient to perceive relative joint position at rest. To formally test proprioception the
therapist holds the joint in a static position and asks the patient to verbally describe the exact position or perhaps
demonstrate the position with the contralateral limb. (O'Sullivan p. 145)

73) A physical therapist employed in an acute care hospital wants to verify that there has not been a medical status
change on an existing patient scheduled to receive therapy services later in the day. The MOST appropriate therapist
action is:
1. contact the patient's nurse
2. examine the patient's medical record
3. page the patient's referring physician
4. ask the patient

Answer: 2
Non-Systems
Safety & Professional Roles; Teaching/Learning; Research
It is relatively common for the medical status of a patient to change significantly during an acute care hospitalization. As a
result, it is critical for therapists to frequently monitor the patient's medical record in an effort to identify any relevant
change in medical status. (Pierson p. 9)

74) A physical therapist works with a 52-year-old female with a wound. The wound is clean, measures one half
centimeter deep, and granulation tissue is just beginning to form. Over a three week period the therapist treats that
patient with a wet-to-dry dressing with acetic acid that is changed twice daily. Which of the following is MOST
likely to occur as a result of this treatment regimen?
1. wound infection
2. hypogranulation with little to no epithelialization
3. hypertrophic or keloid scarring
4. wound healing

Answer:
Integumentary System
Interventions
Wet to dry dressings may result in disruption to the granular bed forming in a wound, and thus, may lead to
hypogranulation and delayed healing. (Kloth p. 212)

Scorebuilders 6
22 Practice Questions

75) A physical therapist examines a posteroanterior x-ray of a patient's left foot using a lateral view. When examining
the x-ray the therapist should identify the bone indicated by the arrow as the:

1. cuboid
2. navicular
3. first cuneiform
4. talus

Answer: 2
Musculoskeletal System
Foundations for Evaluation, Differential Diagnosis, and Prognosis
The navicular is located along the medial aspect of the foot, proximal to the cuneiforms and distal to the talus. (Hoppenfeld
p. 199)

76) A physical therapist receives orders to perform ambulation activities with a female patient status post total knee
replacement. The therapist reads in the patient's medical chart that her hemoglobin value from a recent laboratory
report is 7.8 mg/dL. What is the MOST appropriate level of activity for the patient?
1. no physical activity
2. slow ambulation
3. low-level resistive exercises
4. there is no restriction to physical activity

Answer: 1
Other Systems
Interventions
Individuals with a hemoglobin value of less than 8 mg/dL should not receive or participate in treatment requiring physical
activity. (Pierson p. 286)

77) A physical therapist completes a systems review on a patient immediately after completing a detailed patient history.
Which of the following would NOT typically be included as part of the integumentary component of the systems
review?
1. assessment of skin color
2. assessment of edema
3. presence of scar formation
4. assessment of skin integrity

Answer: 2
Integumentary System
Examination
According to the Guide to Physical Therapist Practice a systems review includes a brief or limited examination of the
anatomical and physiological status of the cardiovascular/pulmonary, integumentary, musculoskeletal, and neuromuscular
systems. The review of the integumentary system includes the assessment of skin integrity, skin color, and the presence of

Scorebuilders 6
Practice Questions 23

scar formation. Edema is a component of the cardiovascular/pulmonary system review. (Guide to Physical Therapist
Practice p. 34)

78) A physical therapist examines a 14-year-old male diagnosed with Osgood-Schlatter disease. The patient reports
limiting the intensity and duration of activities due to a progressive increase in pain over the last month. The MOST
likely objective finding based on the diagnosis is:
1. patella crepitus
2. increased lateral tibial rotation
3. inability to extend the knee against gravity
4. tenderness to palpation over the tibial tubercle

Answer: 4
Musculoskeletal System
Foundations for Evaluation, Differential Diagnosis, and Prognosis
Osgood-Schlatter disease or osteochondrosis of the tibial tuberosity is characterized by swelling and tenderness over the
tibial tubercle that is exacerbated by exercise or any activity requiring active use of the quadriceps. The condition is most
commonly observed in muscular adolescent boys. Patients with Osgood-Schlatter disease often present with an extremely
prominent tibial tubercle. (Magee p. 663)

79) A physical therapist designs an exercise session for a patient diagnosed with a lower motor neuron lesion. The
patient often exhibits a short attention span and when fatigued the therapist has noted a sharp drop off in
performance. What type of practice would be the MOST appropriate based on the patient's current status?
1. massed
2. distributed
3. random
4. blocked

Answer: 2
Neuromuscular and Nervous Systems
Interventions
Distributed practice is characterized by the amount of rest time between trials being equal to or greater than the amount of
time for the trial. (Shumway-Cook p. 41)

80) A 60-year-old male presents with low back pain. The patient describes pain radiating down the outer thigh and leg,
stiffness in the low back region, and increased pain with attempted movement of the spine. Which of the following
would be the MOST likely cause of the patient's symptoms?
1. microtears of the ligamentum flavum and posterior longitudinal ligament
2. L3-L4 lumbar facet joint arthropathy
3. soft tissue strain of the lumbar extensors
4. posterolateral protrusion of the lumbar intervertebral disk at L5-S1

Answer: 4
Neuromuscular and Nervous Systems
Foundations for Evaluation, Differential Diagnosis, and Prognosis
Acute low back pain that radiates down the posterolateral aspect of the thigh and leg is often caused by a posterolateral
protrusion of the lumbar intervertebral disk at the L5-S1 level. The lumbar region of the vertebral column often becomes
rigid and movement is painful due to muscle spasm. (Brotzman p. 572)

81) A physical therapist guards a patient that is descending a flight of stairs with handrails. The patient has left lower
extremity weakness due to a tibial plateau fracture eight weeks ago. Assuming the patient descends the stairs
according to the normal flow of traffic, the MOST appropriate therapist position when guarding the patient is:
1. beside the patient on the involved side
2. beside the patient on the uninvolved side
3. in front of the patient on the uninvolved side
4. in front of the patient on the involved side

Scorebuilders 6
24 Practice Questions

Answer: 4
Non-Systems
Safety & Professional Roles; Teaching/Learning; Research
The standard guarding position when descending stairs is to stand in front of the patient in the area where there is least
protection (usually considered to be the weaker or involved side). Since the patient is descending the stairs according to the
normal traffic flow (on the left) and the stairs are equipped with handrails it would be extremely difficult to guard the
patient on the uninvolved side. (Pierson p. 246)

82) A physical therapist works with a patient recently admitted to a rehabilitation hospital. During the initial session the
therapist notes that the patient has significant difficulty with language to the extent that it is extremely difficult for
the patient to convey even basic information. This type of clinical presentation would be MOST consistent with
1. a 63-year-old female status post right CVA
2. a 41-year-old female diagnosed with Guillain Barre syndrome
3. a 32-year-old male with a traumatic brain injury presently classified as confused-appropriate
4. a 52-year-old male status post left CVA

Answer: 4
Neuromuscular and Nervous Systems
Foundations for Evaluation, Differential Diagnosis, and Prognosis
Patients status post CVA involving the left hemisphere often have significant difficulty with language resulting in various
forms of aphasia. A patient with a traumatic brain injury at the confused-appropriate level may generate responses that may
be incorrect due to memory, but are appropriate for the situation. (O'Sullivan p. 534)

83) As part of a pre-participation health screening for a moderate intensity exercise program, a physical therapist
recognizes the need to classify patients who have more than two risk factors or more than one sign and symptom of
cardiopulmonary disease as "increased risk." This measurement will have predictive validity if individuals classified
as "increased risk" go on to:
1. have a graded exercise test
2. become sedentary
3. stop smoking
4. have a myocardial infarction

Answer: 4
Non-Systems
Safety & Professional Roles; Teaching/Learning; Research
Predictive validity exists when an interpretation is justified by comparing a measurement with supporting evidence
obtained at a later point in time. (Guide to Physical Therapist Practice p. 53)

84) A physical therapist works with a patient who has a right transtibial and a left transfemoral amputation. The patient
is learning to walk up and down stairs with bilateral prostheses. When teaching the patient, the physical therapist
should instruct the patient to lead with the:
1. transtibial prosthesis when ascending and descending
2. transfemoral prosthesis when ascending and descending
3. transtibial prosthesis when ascending, and the transfemoral prosthesis when descending
4. transfemoral prosthesis when ascending, and the transtibial prosthesis when descending

Answer: 3
Musculoskeletal System
Interventions
Due to the presence of the quadriceps muscle, the patient should ascend with the transtibial prosthesis. Although patients
may eventually be able to descend using a step over step pattern it is much easier to descend leading with the transfemoral
prosthesis. (Seymour p. 243)

Scorebuilders 6
Practice Questions 25

85) A physical therapist employed in an acute care hospital takes the resting heart rate of a 30-year-old male. The
therapist records the patient's resting heart rate as 95 beats per minute. The MOST appropriate action to take prior to
initiating treatment would be to:
1. notify the patient's nurse
2. notify the referring physician
3. retake the patient's heart rate
4. review the patient's medical record to determine the patient's baseline heart rate

Answer: 4
Cardiac, Vascular, and Pulmonary Systems
Interventions
It is desirable to review the patient's baseline measurements in order to ascertain whether or not the measured value is
indicative of an abnormal response. Baseline measurements are also critical when determining a patient's response to
activity or exercise. (Pierson p. 48)

86) A physical therapist has been given an assignment by his supervisor to determine the effectiveness of ultrasound for
musculoskeletal pathologies. When searching the available databases, which type of research design would provide
the highest level of evidence?
1. systematic review of randomized controlled trials
2. individual randomized control with a large number of subjects
3. systematic review of case-control studies
4. systematic review of cohort studies

Answer: 1
Non-Systems
Safety & Professional Roles; Teaching/Learning; Research
A systematic review of randomized controlled trials affords the highest level of evidence for evidence-based practice.
(Sackett)

87) A physical therapist instructs a physical therapist assistant to perform neuromuscular electrical stimulation to the
wrist extensors of a patient rehabilitating from a wrist injury. After setting up the patient according to the specified
parameters provided by the physical therapist, the physical therapist assistant believes the muscle contraction
produces too much radial deviation and insufficient wrist extension. The MOST appropriate physical therapist
assistant action is:
1. increase the current intensity
2. reposition the electrodes
3. alter the wave form of the current
4. discuss the situation with the supervising physical therapist

Answer: 2
Non-Systems
Safety & Professional Roles; Teaching/Learning; Research
A physical therapist assistant can modify the parameters of an existing intervention within the established plan of care.
Repositioning the electrodes would be the most appropriate option to better recruit the necessary muscles and produce the
desired action. (Cameron p. 242)

88) A physical therapist administers an ultrasound treatment using water immersion on a patient rehabilitating from
ankle surgery. During the session the therapist notices that air bubbles tend to accumulate on the sonated area of the
skin and face of the soundhead. The MOST appropriate therapist action is:
1. wipe off the air bubbles on the skin periodically
2. wipe off the air bubbles on the face of the soundhead periodically
3. wipe off the air bubbles on the skin and face of the soundhead periodically
4. avoid wiping off any of the air bubbles

Answer: 3
Non-Systems

Scorebuilders 6
26 Practice Questions

Equipment & Devices; Therapeutic Modalities


The air bubbles on the skin and the face of the soundhead should be periodically removed to promote maximum
transmission of the ultrasound energy. (Belanger p. 235)

89) A physical therapist demonstrates a D2 flexion proprioceptive neuromuscular facilitation pattern to a patient
rehabilitating from an upper extremity injury. As the patient begins the activity the therapist quickly concludes the
patient is growing increasingly confused. Which verbal instruction would be the MOST appropriate to assist the
patient to complete the activity correctly?
1. open your hand and push down and away from your body
2. open your hand and pull up and away from your body
3. close your hand and pull up and across your body
4. close your hand and pull down and across your body

Answer: 2
Neuromuscular and Nervous Systems
Interventions
A D2 flexion pattern incorporates flexion, abduction, and lateral rotation of the shoulder and therefore promotes upright
posture. The verbal command to complete this pattern would be open your hand and pull up and away from your body.
(Kisner p. 117)

90) A physical therapist employed in an outpatient physical therapy clinic examines a patient recently referred to
physical therapy. While completing the examination the therapist identifies a significantly diminished Achilles
tendon reflex. Which of the following patients would MOST likely present with a diminished or absent Achilles
tendon reflex?
1. a 20-year-old basketball player with a history of patellar tendonitis
2. a significantly obese 42-year-old female with systemic lupus erythematosus
3. a 60-year-old male with diabetes mellitus and polyneuropathy
4. a 65-year-old female with peripheral vascular disease

Answer: 3
Neuromuscular and Nervous Systems
Foundations for Evaluation, Differential Diagnosis, and Prognosis
Muscle spindles in the plantar flexors are stimulated when the Achilles tendon is tapped, sending afferent impulses to the
spinal cord. Efferent impulses are sent back to the plantar flexors via motor fibers resulting in a jerk-like contraction of the
plantar flexors and spontaneous plantar flexion of the ankle. Diminution or absence of this reflex may result from any
lesion that interrupts this reflex arc such as a diabetic neuropathy. (Moore p. 393)

91) A physical therapist reviews the medical record of a patient recently admitted to physical therapy after being
diagnosed with adhesive capsulitis. The patient describes an insidious onset of pain and stiffness approximately three
months ago. Which patient profile would be MOST likely based on the medical diagnosis?
1. a 61-year-old female with multiple sclerosis
2. a 45-year-old male with peripheral neuropathy
3. a 52-year-old male with a history of recurrent rotator cuff tendonitis
4. a 48-year-old female with diabetes

Answer: 4
Other Systems
Foundations for Evaluation, Differential Diagnosis, and Prognosis
Adhesive capsulitis is characterized by the loss of motion at the glenohumeral joint followed by a gradual stiffness. The
condition is most common in patients from 40 to 60 years of age with a significantly higher incidence in females than
males. Patients with insulin dependent diabetes are particularly susceptible to adhesive capsulitis and in some cases actually
experience bilateral involvement. (Brotzman p. 227)

92) A physical therapist examines a patient diagnosed with "whiplash" following a motor vehicle accident. The accident
occurred approximately two months ago. Prior to initiating a trial of manual traction the therapist attempts to

Scorebuilders 6
Practice Questions 27

determine if the patient is an appropriate candidate. Which of the following findings would be considered a
contraindication for the specified intervention?
1. rheumatoid arthritis
2. cervical hypomobility
3. headaches
4. neck pain

Answer: 1
Non-Systems
Equipment & Devices; Therapeutic Modalities
Rheumatoid arthritis could cause instability of the alar and/or transverse ligaments, and as a result traction could potentially
be detrimental. (Kisner p. 623)

93) A physical therapist reviews the results of a research study that examined the reliability of an electrogoniometer.
Which reliability coefficient would be MOST desirable from the therapist's perspective?
1. 0.09
2. 0.37
3. 0.63
4. 0.91

Answer: 4
Non-Systems
Safety & Professional Roles; Teaching/Learning; Research
Reliability refers to the degree of consistency of the electrogoniometer. As a result, the reliability coefficient that most
closely approximates 1.0 (i.e., perfectly reliable) would be the most desirable for the therapist. (Portney p. 65)

94) A physical therapist is working with a patient who recently had a left middle cerebral artery stroke. Currently, the
patient demonstrates minimal active movement on the right side and is having extreme difficulty moving in bed.
Which of the following strategies will be MOST useful when initially teaching the patient to roll supine to left
sidelying?
1. provide detailed verbal cues
2. use part-practice
3. avoid knowledge of performance feedback
4. provide feedback on 100% of the practice trials

Answer: 2
Neuromuscular and Nervous Systems
Interventions
The use of part-practice may be effective in improving the patient's learning since the patient is beginning to relearn a task
that is comprised of a number of steps (rolling to sidelying). Given that the patient has had a left middle cerebral artery
stroke, speech and language difficulties are likely so detailed verbal cues should be avoided. Feedback regarding
knowledge of performance may be useful initially to give the patient an idea of the movement components of the task.
Feedback on 100% of trials may interfere with learning. (Shumway-Cook p. 43)

95) A physical therapist talks with a mother holding her three-month-old infant. During the conversation the mother
loses control of the baby's head, suddenly dropping it into extension. What reflex would this action MOST likely
elicit?
1. Moro
2. grasp
3. tonic labyrinthine
4. extensor thrust

Answer: 1
Neuromuscular and Nervous Systems
Examination

Scorebuilders 6
28 Practice Questions

The Moro reflex is elicited secondary to a sudden change in position of the head in relation to the trunk. The response
begins at 28 weeks of gestation and is typically integrated between five and six months of age. (O'Sullivan p. 187)

96) A 25-year-old male immigrant with low back pain is referred to physical therapy. After completing the examination
the physical therapist recommends ongoing physical therapy services. The patient politely declines the
recommendation and states that he will be using traditional herbal remedies. The MOST appropriate therapist action
is to:
1. listen respectfully to the patient, then provide him with a detailed explanation to help him understand the correct
treatment for the condition
2. listen carefully to the patient and if it becomes clear that he is committed to his beliefs, inform him that you will be
unable to assist him
3. inform the patient that you cannot assist him and refer the patient to a traditional healer who will provide
alternative models of care
4. listen to the patient's explanation of his problem, provide an explanation from a physical therapy perspective, and
negotiate a program that is acceptable for the patient

Answer: 4
Non-Systems
Safety & Professional Roles; Teaching/Learning; Research
An explanatory model of care is a concept that provides a foundation for understanding the patient and provider's belief
systems regarding causes of health or illness. The most appropriate method to communicate with the patient in order to
bridge the gap regarding treatment would be to identify the patient's beliefs, explain your own, and negotiate the differences
to provide the best possible care. (Shepard p. 329)

97) A physical therapist employed in an acute care hospital discusses the plan of care for a patient rehabilitating from a
fractured pelvis with the referring physician. During the discussion the physician indicated the fracture should be
stable enough for the patient to begin lower extremity resistive exercises. The following day the therapist checks the
patient's medical record and does not locate a new order from the physician. The therapist attempts to contact the
physician's office, however is unable to reach the physician or the physician's nurse. The MOST appropriate therapist
action is:
1. place the patient on hold until the orders are clarified with the physician
2. discuss the situation with another physician on site
3. continue to treat the patient based on the previous orders
4. introduce lower extremity resistive exercises

Answer: 3
Non-Systems
Safety & Professional Roles; Teaching/Learning; Research
The physical therapist likely contacted the physician's office in order to obtain a formal order to modify the patient's plan of
care. As a result it would be unacceptable for the physical therapist to begin lower extremity resistive exercises after being
unable to clarify the physician's informal request. Although the therapist is unable to begin resistive exercises with the
patient, it would be desirable to continue to treat the patient based on the previous orders. (Guide to Physical Therapist
Practice p. 39)

98) A physical therapist designs a general strengthening program for a male patient rehabilitating from a flexor tendon
repair. The therapist instructs the patient to place the rubber band around his fingers when they are together and then
asks the patient to move the fingers as far apart as possible. The PRIMARY objective of this activity is to:

Scorebuilders 6
Practice Questions 29

1. stretch the lumbricales


2. stretch the interossei
3. strengthen the lumbricales
4. strengthen the interossei

Answer: 4
Musculoskeletal System
Interventions
The dorsal interossei act to abduct the index, middle, and ring fingers from the axial line through the third digit. (Kendall p.
248)

99) A physical therapist administers an ultrasound treatment to the low back of a patient recently referred to physical
therapy. During the treatment the patient suddenly complains of "hot spots" directly under the ultrasound transducer.
The ultrasound parameters MOST likely to result in this type of subjective complaint are:
1. stationary technique, high beam nonuniformity ratio
2. dynamic technique, low beam nonuniformity ratio
3. stationary technique, low beam nonuniformity ratio
4. dynamic technique, high beam nonuniformity ratio

Answer: 1
Non-Systems
Equipment & Devices; Therapeutic Modalities
A stationary technique combined with a high beam nonuniformity ratio would be most likely to produce hot spots under the
soundhead. A high beam nonuniformity ratio produces relatively high peaks of energy. (Belanger p. 227)

100) A 65-year-old male status post right total hip arthroplasty complains of right lower extremity pain. The physical
therapist determines that the pain is localized to the right gastrocnemius region and becomes significantly worse with
forceful passive ankle dorsiflexion and squeezing of the gastrocnemius muscle. Which of the following is the MOST
likely cause of the discomfort?
1. soft tissue strain of the gastrocnemius muscle
2. normal postoperative pain
3. acute deep vein thrombophlebitis
4. contracture of the gastrocnemius muscle

Answer: 3
Other Systems
Foundations for Evaluation, Differential Diagnosis, and Prognosis
Homans' sign consists of passive dorsiflexion of the foot with the knee in an extended position. The test is designed to
identify the presence of a deep vein thrombosis and is considered positive if the patient experiences pain in the calf. (Magee
p. 808)

Scorebuilders 6
30 Practice Questions

101) As part of a reassessment for a patient with a Colles' fracture a physical therapist measures range of motion with a
goniometer. The therapist positions the patient in sitting with the involved extremity supported on a treatment plinth.
To measure wrist ulnar deviation, the movable arm of the goniometer should be aligned with the
1. third metacarpal
2. capitate
3. triquetrum
4. radial styloid process

Answer: 1
Musculoskeletal System
Examination
When measuring wrist ulnar deviation, the hand is placed palm down on a firm surface. The fulcrum is placed over the
capitate, the stationary arm is placed along the dorsal midline of the forearm using the lateral epicondyle as a reference, and
the movable arm is positioned along the dorsal midline of the third metacarpal. (Reese p. 104)

102) A physical therapist examines a patient referred to physical therapy with a prescription that reads "evaluate and
treat - status post left knee sprain with resultant left lower extremity weakness." While taking a patient history, the
therapist identifies complaints of lower extremity weakness, severe facial pain, vertigo, visual disturbance,
paresthesias in the lower extremities, fatigue, and urinary bladder disturbances. These findings are MOST consistent
with:
1. Lyme disease
2. Parkinson's disease
3. multiple sclerosis
4. rheumatoid arthritis

Answer: 3
Neuromuscular and Nervous Systems
Foundations for Evaluation, Differential Diagnosis, and Prognosis
Multiple sclerosis is a chronic progressive demyelinating disease characterized by exacerbations of symptoms followed by
remissions. The course and progression of the disease is relatively unpredictable, however early symptoms include visual
impairments and paresthesias. (Goodman - Differential Diagnosis p. 402)

103) A numeric rating scale of pain has been shown to have an intraclass correlation coefficient (ICC) of 0.97 when
administered to patients with arthritis twice within the same day. What can be concluded about the pain scale from
this statistic?
1. The scale is valid when it is administered twice within the same day.
2. Pain is measured on the ordinal scale of measurement
3. The scale has good test-retest reliability when administered twice within the same day.
4. The pain scale is clinically significant.

Answer 3
Non-Systems
Safety & Professional Roles; Teaching/Learning; Research
The ICC is a reliability coefficient that reflects both degree of correspondence and agreement among ratings. In the
example, two sets of pain ratings were made on the same day so the ICC is used to assess test-retest reliability. (Portney p.
560)

104) A physical therapist administers Craig's test to a 14-year-old female in order to quantify the amount of anteversion
or retroversion in the patient's right hip. After completing the test the therapist asks the patient to maintain the
position while he retrieves a goniometer. The measurement MOST consistent with the presented image would be:

Scorebuilders 6
Practice Questions 31

1. 5 degrees of anteversion
2. 20 degrees of anteversion
3. 5 degrees of retroversion
4. 20 degrees of retroversion

Answer: 2
Musculoskeletal System
Examination
Anteversion of the hip is measured by the angle made by the femoral neck with the femoral condyles. It can be quantified
by using a goniometer with the axis on the midpoint of the patella, the stationary arm perpendicular with the floor, and the
moving arm aligned with the anterior midline of the shaft of the tibia. 30 degrees of anteversion is normal at birth, however
the amount of anteversion gradually decreases to 8-15 by the time an individual becomes an adult. (Magee p. 621)

105) A physical therapist employed in an acute care hospital left an 85-year-old female sitting unguarded on the edge of
her bed while he collected the patient's slippers and robe. While unattended, the patient fell and fractured her hip.
The patient's MOST likely cause of action against the therapist is:
1. negligence
2. felony
3. breach of contract
4. malpractice

Answer: 1
Non-Systems
Safety & Professional Roles; Teaching/Learning; Research
Negligence is defined as a violation of the required standard of care. The standard of care in this situation would be to
collect the slippers, robe, and other necessary implements prior to having the patient sit on the edge of the bed. (Scott-
Health Care Malpractice p. 16)

106) In busy health care settings it is common to hear professionals refer to patients as "amputees", "CVAs" or "total
knees." Such terminology is:
1. acceptable since it helps to identify the patient's health related problem
2. acceptable because patients are educated as to their medical diagnosis
3. unacceptable because it de-humanizes the patient
4. unacceptable because the patient might have multiple diagnoses

Answer: 3
Non-Systems
Safety & Professional Roles; Teaching/Learning; Research
People first language is the accepted norm when referring to patients or clients. This type of language places the person
before the disability and enhances the provider's ability to see the person, not just the medical condition. (Drench p. 6)

107) A physical therapist employed in a rehabilitation hospital treats a 34-year-old female patient diagnosed with
amyotrophic lateral sclerosis. The patient was diagnosed with the disease only four months ago, however has
experienced a rapid decline in her ability to function independently. During the treatment session the patient asks
how long she will live. The MOST appropriate therapist response is:

Scorebuilders 6
32 Practice Questions

1. explain to the patient you are limited in your ability to interpret the medical record
2. answer the patient's question based on your knowledge of amyotrophic lateral sclerosis
3. discuss the patient's current functional limitations based on amyotrophic lateral sclerosis
4. encourage the patient to speak directly with the physician

Answer: 4
Non-Systems
Safety & Professional Roles; Teaching/Learning; Research
The physical therapist needs to respond to the question directly, however, given the sensitivity of the question and the
obvious medical implications it is best to encourage the patient to speak to her referring physician. Although the therapist
may be able to provide an estimate of the patient's remaining time based on the disease progression, the physician is in a
better position to make a more realistic estimate. Discussing the patient's functional limitations would be appropriate, but
does not address the patient's question directly. (Guide to Physical Therapist Practice p. 39)

108) A physical therapist initiates ultrasound using water immersion to the distal calf of a patient rehabilitating from a
motor vehicle accident. The therapist had previously administered ultrasound using direct contact; 1 MHz frequency
at 1.2 W/cm2 for five minutes. In order to maintain the same amount of tissue heating as originally used with the
direct contact technique the therapist should:
1. utilize the same treatment procedures
2. increase the intensity and decrease the time
3. increase the intensity and increase the time
4. change the frequency to 3 MHz and increase the time

Non-Systems
Equipment & Devices; Therapeutic Modalities
The water immersion technique requires an increase in intensity and/or an increase in time in order to produce the same
heating effects as the direct contact technique. The modifications are necessary to compensate for the thermal energy loss to
water. (Belanger p. 247)

109) Which of the following category of drugs is used in the management of heart failure because it decreases preload?
1. calcium-channel blockers
2. sympathomimetics
3. cardiac glycosides
4. diuretics

Answer: 4
Cardiac, Vascular, and Pulmonary Systems
Clinical Application of Foundational Sciences
Diuretics decrease circulating blood volume, thereby decreasing preload. Diuretics encourage the excretion of urine
(diuresis) and influence water and electrolyte balance by inhibiting sodium and water reabsorption. The most common
diuretic for symptomatic heart failure is the loop diuretic furosemide (Lasix), which, in addition to sodium inhibition, also
inhibits the movement of potassium and chloride across Henle's loop in the kidneys. Calcium-channel blockers,
sympathomimetics, and cardiac glycosides also are used in the treatment of heart failure, but have different effects.
Calcium-channel blockers work by relaxing smooth muscle within the arterial walls, resulting in decreased afterload.
Sympathomimetics and cardiac glycosides increase cardiac output by increasing myocardial contractility. (Hillegass p. 551)

110) A physical therapist works with a patient who is recovering from a traumatic T5 spinal cord injury. During the
session the patient becomes extremely restless, begins to sweat, and complains of a severe headache. The therapist
measures the patient's blood pressure as 200/120 mm Hg. Based on the supplied information, the patient is MOST
likely experiencing:
1. cerebrovascular accident
2. acute anxiety attack
3. postural hypotension
4. autonomic dysreflexia

Answer: 4
Neuromuscular and Nervous Systems

Scorebuilders 6
Practice Questions 33

Interventions
Autonomic dysreflexia is a pathologic autonomic reflex that occurs in lesions above the T6 level. Symptoms of autonomic
dysreflexia include profuse sweating, hypertension, severe pounding headache, blurred vision, and restlessness. (O'Sullivan
p. 883)

111) A physical therapist takes a series of measurements in order to secure an appropriate wheelchair for a patient
recently admitted to a rehabilitation hospital. When determining seat depth the MOST appropriate formula to utilize
is:
1. heel to popliteal crease plus two inches
2. posterior buttocks to popliteal crease minus two inches
3. seat to axilla minus four inches
4. widest part of buttocks plus two inches

Answer: 2
Non-Systems
Equipment & Devices; Therapeutic Modalities
Seat depth is measured from the posterior buttocks along the lateral thigh to the popliteal crease. Two inches should be
subtracted from the measurement to prevent pressure in the popliteal space. (Pierson p. 168)

112) While evaluating a patient diagnosed with cervical radiculopathy, a physical therapist discovers that a patient
cannot differentiate between sharp and dull on the tip of his right thumb. Based on this information, the therapist
suspects involvement of the
1. C4 dermatome
2. C5 dermatome
3. C6 dermatome
4. C7 dermatome

Answer: 3
Neuromuscular and Nervous Systems
Foundations for Evaluation, Differential Diagnosis, and Prognosis
The C6 dermatome extends from approximately the lateral forearm to the second finger, encompassing the thumb. Inability
to differentiate sharp and dull to this region may indicate C6 nerve root or disk pathology. (Hoppenfeld p. 118)

113) A physical therapist administers an electrotherapy treatment for the purpose of muscle reeducation. According to
the strength duration curve, which of the following BEST describes the order of nerve fiber recruitment as the
amplitude is increased?
1. motor nerve fibers, sensory nerve fibers, pain fibers
2. pain nerve fibers, motor nerve fibers, sensory nerve fibers
3. sensory nerve fibers, motor nerve fibers, pain nerve fibers
4. motor nerve fibers, pain nerve fibers, sensory nerve fibers

Answer: 3
Non-Systems
Equipment & Devices; Therapeutic Modalities
Sensory nerves can be depolarized with lower current amplitudes and shorter pulse durations than motor nerve fibers.
Similarly motor nerve fibers can be depolarized with lower current amplitudes and shorter pulse durations than pain nerve
fibers. (Cameron p. 228)

114) A female patient who is status post stroke is working on ambulation in physical therapy. The patient has good
sensation and her strength is within functional limits. When walking in the community, the patient is able to walk up
stairs with supervision. Yet, when the patient practices this task in the therapy gym, the patient is unable to complete
the activity. This is MOST likely a problem of:
1. ideomotor apraxia
2. impaired body schema
3. impaired problem solving

Scorebuilders 6
34 Practice Questions

4. ideational apraxia

Answer: 1
Neuromuscular and Nervous Systems
Foundations for Evaluation, Differential Diagnosis, and Prognosis
The patient has sufficient motor and sensory ability to perform the task, but is unable to do so. An ideomotor apraxia is one
in which the patient can complete the task automatically, but not on command. In this case, the patient is able to perform
the task of stair climbing when encountered in a typical environment, but is unable to do so when in the therapy gym. This
type of scenario is most consistent with ideomotor apraxia. An ideational apraxia results in the patient being unable to
perform the task under either circumstance. (Shumway-Cook p. 158)

115) A physical therapist completes an assessment on a competent patient, announces to the patient his choice of
treatment, and immediately begins a course of therapy. If the therapist initiates treatment immediately, from a legal
perspective the therapist has failed to:
1. discuss the risks of treatment
2. discuss the benefits of treatment
3. answer the patient's questions
4. obtain informed consent

Answer: 4
Non-Systems
Safety & Professional Roles; Teaching/Learning; Research
Informed consent includes discussing the risks and benefits of treatment and answering patient questions. The patient is
then required to consent to the treatment plan based on information that they received from the therapist prior to the
initiation of any therapeutic intervention. (Scott-Health Care Malpractice p. 39)

116) A physical therapist treats a 73-year-old female status post CVA. The patient complains of fatigue, lack of interest
in activities, and the inability to concentrate or remember things. The patient has difficulty performing therapeutic
activities and has no desire to perform her home exercise program since she believes her condition is hopeless. The
MOST likely explanation is:
1. the patient is quite content with her functional level
2. the patient is exhibiting signs and symptoms of depression
3. the description of the patient is consistent with the majority of geriatric patients
4. the patient does not understand the potential benefits of physical therapy

Answer: 2
Non-Systems
Safety & Professional Roles; Teaching/Learning; Research
Depression is a common condition and is seen in many individuals with various health problems. Symptoms of depression
include fatigue, lack of interest in previously enjoyable activities, inability to concentrate or remember, and hopelessness.
Depression is a condition that can be effectively treated by a variety of different strategies. (Goodman - Pathology p. 55)

117) A physical therapist working in an outpatient clinic examines a patient referred to physical therapy following
complications from a total hip replacement. During the session the patient asks the therapist to interpret numerous
passages from an operative report. The MOST appropriate therapist action is:
1. interpret the operative report for the patient
2. explain to the patient that the physician would be the most appropriate health care provider to interpret the
operative report
3. inform the patient that the medical terminology makes interpreting the report too difficult to explain
4. refer the patient to the director of rehabilitation

Answer: 2
Non-Systems
Safety & Professional Roles; Teaching/Learning; Research

Scorebuilders 6
Practice Questions 35

The physician would be the most appropriate individual to interpret the operative report since they were the author and
actually performed the surgery. By attempting to interpret the report the physical therapist may be allowing for unnecessary
legal risk and perhaps jeopardizing their relationship with the physician. (Standards of Practice)

118) A patient who received iontophoresis during his last physical therapy session indicates that his skin under one of
the electrodes stayed red for approximately two hours after the treatment. Documentation from the session indicates
that the therapist used a current amplitude of 3.0 milliamperes for 10 minutes. Based on the patient's subjective
report the therapist should:
1. continue with the present treatment
2. decrease the current amplitude
3. decrease the treatment duration
4. select a different therapeutic ion

Answer: 1
Non-Systems
Equipment & Devices; Therapeutic Modalities
Skin irritation under an electrode that remains for several hours after treatment is relatively common when using
iontophoresis. As a result, the therapist should continue with the established parameters. Skin irritation under the electrodes
can be minimized by cleaning the skin surface with isopropyl alcohol prior to beginning treatment. (Belanger p. 17)

119) A physical therapist employed in an acute care hospital treats a 67-year-old male with a venous ulcer immediately
posterior to the medial malleolus. The patient has a lengthy medical history of venous insufficiency including
admission to the hospital six months ago for a deep venous thrombosis. Which finding is MOST likely based on the
patient's present condition?
1. pale, white skin color
2. absent peripheral pulses
3. diminished pain with the legs elevated
4. thin, shiny skin with hair loss

Answer: 3
Integumentary System
Foundations for Evaluation, Differential Diagnosis, and Prognosis
Elevation of the legs assists with venous return and therefore would tend to be more comfortable for a patient with venous
insufficiency. The remaining options are signs and symptoms of arterial insufficiency. (Goodman - Pathology p. 463)

120) A 19-year-old male rehabilitating from a tibia fracture sustained in a motor vehicle accident is referred to physical
therapy. The patient demonstrates loss of active dorsiflexion and a high stepping gait in which he raises his foot
higher than necessary and suddenly brings it down producing a "slapping" sound. What is the MOST likely rationale
for the patient's current condition?
1. severance of the common peroneal nerve
2. anterior compartment syndrome
3. hip flexor weakness
4. calcaneal spurring

Answer: 1
Other Systems
Foundations for Evaluation, Differential Diagnosis, and Prognosis
Severance of the common peroneal nerve causes paralysis of the dorsiflexor muscles and results in a high stepping gait. The
common peroneal nerve is the most commonly injured nerve in the lower extremity due to its superficial position as it
wraps around the neck of the fibula. (Hall p. 493)

121) A physical therapist presents a lecture at a community recreation center on initiating a muscle strengthening
program. During the session the physical therapist is asked how to prevent the muscle soreness that often results
from strength training. Which type of exercise would create the MOST muscle soreness?
1. isometric
2. concentric

Scorebuilders 6
36 Practice Questions

3. eccentric
4. isokinetic

Answer: 3
Musculoskeletal System
Interventions
Eccentric exercise often incorporates increased resistance (load) and is therefore associated with a greater incidence of
delayed onset of muscle soreness. Muscle soreness from eccentric exercise usually begins 12-24 hours after exercise and in
severe cases can last 10-14 days. (Kisner p. 100)

122) A physical therapist works with a patient who sustained a closed tibia fracture. The patient was non-weight
bearing and casted for six weeks prior to being referred to physical therapy. As part of a treatment session the
therapist explains how resistance training will increase the patient's knee extensor strength. From a physiological
perspective the change in strength is due to:
1. hypertrophy of muscle fibers causing an increase in the cross sectional area of the muscle
2. increase in the number of active muscle fibers
3. splitting of current muscle fibers
4. increase in bone density of the femur and tibia

Answer: 1
Musculoskeletal System
Interventions
Resistance training results in hypertrophy of muscle fibers and increased recruitment of motor units. As a result the muscle
is able to generate
an increased amount of force. (Tan p. 161)

123) A physical therapist administers an electrotherapy intervention using the following parameters; frequency of 100-
150 pulses per second, phase duration greater than 300 microseconds, intensity to noxious level, and on/off ratio of
10 seconds on, 5 seconds off. Which of the following BEST describes the electrotherapy treatment being delivered?
1. motor level electroanalgesia
2. brief intense transcutaneous electrical nerve stimulation
3. sensory level electroanalgesia
4. low frequency transcutaneous electrical nerve stimulation

Answer: 2
Non-Systems
Equipment & Devices; Therapeutic Modalities
Brief intense transcutaneous electrical nerve stimulation is characterized by a long pulse duration (greater than 150
microseconds) and a high frequency (greater than 80 pulses per second). (Belanger p. 31)

124) A 21-year-old male has a complete spinal cord injury at the L2 level. The patient is independent with wheelchair
use and has returned to independent living in the home and community. He is otherwise healthy and expresses a
desire to begin gait training. The goal of the gait training is to enable the patient to independently:
1. stand daily, in a standing frame for weight bearing
2. ambulate within his home environment with knee-ankle-foot orthoses
3. ambulate within the home and community with knee-ankle-foot orthoses
4. ambulate within the home and community with ankle-foot orthoses

Answer: 2
Neuromuscular and Nervous Systems
Interventions
Since the patient has a complete spinal cord injury at the L2 level he will require bilateral knee-ankle-foot orthoses
(KAFOs) for ambulation. Household ambulation is the most realistic goal. Ambulation with KAFOs is extremely energy
consuming, thus the patient would be most efficient moving around the community at a wheelchair level. (Umphred p. 523)

Scorebuilders 6
Practice Questions 37

125) A physical therapy director develops a formal objective related to the clinic's documentation for new hires. The
objective states "following a discussion and demonstration with the department director, the therapist will correctly
enter daily charges into the computer." An analysis of the identified objective reveals that:
1. the objective should be rewritten to include a condition
2. the objective should be rewritten to include a degree
3. the objective should be rewritten to include a behavior
4. the objective is correctly written

Answer: 4
Non-Systems
Safety & Professional Roles; Teaching/Learning; Research
The objective contains all four components of a well-written objective: audience, behavior, condition, and degree. (Shepard
p. 60)

126) A physical therapist employed in an inpatient rehabilitation facility notices a colleague often has difficulty
motivating her patients during treatment sessions. The BEST strategy to increase the patient's motivation and
commitment is:
1. share personal information with the patient since this will strengthen the therapist-patient bond
2. tell jokes to create an environment that is conducive to learning
3. frequently correct the patient's performance in order to direct the post-discharge goals and lifestyle
4. treat patients with warmth and frequently ask for their input regarding their care

Answer: 4
Non-Systems
Safety & Professional Roles; Teaching/Learning; Research
The last behavior demonstrates a sense of empathy and caring that promotes a healing relationship and allows the patient to
possess some control over the process. (Purtilo p. 158)

127) A physical therapist and a physical therapist assistant employed in a rehabilitation hospital work with a patient
diagnosed with a spinal cord injury. The patient is scheduled to be discharged from the hospital in three weeks and as
a result the treatment program begins to focus on elements associated with discharge. Which statement BEST
describes the appropriate role of the two health care providers?
1. have the physical therapist assistant participate in discharge activities, however have the physical therapist
complete the discharge summary
2. have the physical therapist assistant participate in discharge activities and complete the discharge summary
3. have the physical therapist complete discharge activities, however have the physical therapist assistant complete
the discharge summary
4. have the physical therapist complete all activities associated with discharge including the discharge summary

Answer: 1
Non-Systems
Safety & Professional Roles; Teaching/Learning; Research
Physical therapist assistants routinely participate in discharge activities, however the physical therapist is the only provider
that should complete a discharge summary at the conclusion of physical therapy services. (Guide to Physical Therapist
Practice p. 42)

128) A physical therapist participates in a community health fair by screening elderly patients to determine their risk for
falling. The therapist utilizes the Berg Balance Scale for the screening and therefore is required to rate patients on 14
specific items. When using the Berg Balance Scale each of the items should be scored using a
1. three point scale
2. four point scale
3. five point scale
4. six point scale

Answer: 3
Non-Systems
Safety & Professional Roles; Teaching/Learning; Research

Scorebuilders 6
38 Practice Questions

The Berg Balance Scale consists of 14 tasks of every day life that are scored from a 0-4 (five point) scale. A score of 0
represents the inability to complete the task and a score of 4 indicates independence. The maximum total score possible
using the Berg is 56, with a score of less than 45 indicating the patient is at risk for multiple falls. (Physical Therapist's
Clinical Companion p. 105)

129) A physical therapist treats a patient referred to physical therapy after being diagnosed with arteriosclerosis
obliterans. The therapist begins a walking program with the patient on a treadmill. After two minutes of walking at
1.0 mile per hour the patient reports significant cramping and pain in her lower legs. The MOST appropriate
therapist action is to:
1. encourage the patient to walk through the pain unless it becomes unbearable
2. decrease the treadmill speed by one-half mile per hour and instruct the patient to continue walking
3. allow the patient to rest and resume walking when the pain subsides
4. discontinue walking and select an alternate exercise

Answer: 3
Cardiac, Vascular, and Pulmonary Systems
Interventions
A progressive ambulation program can be extremely beneficial for patients with peripheral arterial disease. During the
initial stages of the program the therapist should instruct the patient to rest when they experience signs and symptoms of
claudication. As the patient becomes more familiar with the program the therapist may elect to have the patient continue
walking through tolerable levels of pain. (Goodman - Pathology p. 455)

130) A 45-year-old female that has recovered from an extended illness is referred to physical therapy for home exercise
program instruction. Which of the following accurately describes the benefits of aerobic exercise?
1. increase in stroke volume, decrease in blood pressure, and decrease in blood cholesterol
2. decrease in resting pulse rate, increase in triglycerides, and increase in cardiac output
3. increase in resting pulse rate and decrease in exercise threshold for accumulation of lactate in the blood
4. increase in high density lipoprotein cholesterol, decrease in body fat, and decrease in stroke volume

Answer: 1
Cardiac, Vascular, and Pulmonary Systems
Clinical Application of Foundational Sciences
Aerobic exercise produces cardiovascular, respiratory, and metabolic benefits that include a decrease in heart rate, blood
pressure, cholesterol, body fat, and insulin requirements. Aerobic exercise improves cardiac contractility, cardiac output,
high density lipoprotein cholesterol, lung volumes, and ventilatory efficiency. (Kisner p. 162)

131) A physical therapist receives orders for a trial of serial casting for a patient rehabilitating from a traumatic brain
injury. The patient has fluctuating tone in the lower extremities and has a moderate plantar flexion contracture on the
right. The patient is presently classified as confused-agitated using the Rancho Los Amigos Levels of Cognitive
Functioning Scale. What would be the PRIMARY risk associated with the proposed intervention?
1. skin breakdown
2. increased tone
3. muscle atrophy
4. facilitate primitive reflexes

Answer: 1
Non-Systems
Equipment & Devices; Therapeutic Modalities
Serial casting is often used as an intervention for patients with a plantar flexion contracture due to increased tone or
prolonged shortening. The patient's diagnosis of traumatic brain injury makes it possible that the patient has diminished
sensation along with impaired cognition and communication making the risk of skin breakdown even more significant.
(O'Sullivan p. 806)

Scorebuilders 6
Practice Questions 39

132) A physical therapist coordinating a cardiac rehabilitation program educates several patients about the potential for
selected activities to significantly increase blood pressure. Which activity would typically produce the GREATEST
increase in the patient's systolic blood pressure?
1. walking
2. cycling
3. sitting unsupported
4. shoveling

Answer: 4
Cardiac, Vascular, and Pulmonary Systems
Interventions
Exercises which involve upper body movements cause a greater rise in blood pressure than exercises primarily emphasizing
the lower extremities. (Brannon p. 73)

133) A physical therapy plan of care includes ultrasound to the insertion of the biceps femoris tendon. The MOST
appropriate area for the therapist to sonate is:
1. slightly proximal to the fibula head
2. slightly proximal to the pes anserine region
3. slightly proximal to the tibial tuberosity
4. slightly distal to the popliteal fossa

Answer: 1
Musculoskeletal System
Clinical Application of Foundational Sciences
The biceps femoris muscle is considered a lateral hamstring muscle that acts to flex and laterally rotate the knee. The
muscle's primary insertion is on the lateral side of the head of the fibula. (Kendall p. 209)

134) A 28-year-old female with low back pain is referred to physical therapy. The patient indicates that her pain started
the previous weekend after painting and currently is localized on the left side of her low back. The patient states the
pain is better when sitting in a "slouched" posture and is worse during periods of prolonged standing. An
examination reveals a positive quadrant test and positive Kemp's test on the left. Based on the presented information
the MOST likely structures involved are the:
1. right thoracic facets
2. left thoracic facets
3. left lumbar facets
4. right lumbar facets

Answer: 3
Musculoskeletal System
Foundations for Evaluation, Differential Diagnosis, and Prognosis
The lumber facet joints are likely to be stressed during prolonged lumbar extension, especially with a rotation/side flexion
component (painting overhead). Flexion will relieve pressure on these structures, especially if using a "slouched" posture.
Facet irritation is indicated with a positive quadrant test and Kemp's test. The pain on the left side of the low back is
consistent with the left lumbar facets. (Magee p. 525)

135) A female physical therapist completes a learning style inventory that assists the therapist to better understand her
personal learning style preference. How is the information BEST utilized?
1. the therapist should gain awareness about her learning style preferences and attempt to utilize this information to
enhance learning in a variety of different situations
2. the therapist should resort to her preferred learning style preferences when confronted with stressful situations
3. the therapist should seek employment positions that further strengthen the identified areas of preference
4. the therapist should refuse assignments that will challenge her identified weaker areas of learning preferences

Answer: 1
Non-System
Safety & Professional Roles; Teaching/Learning; Research

Scorebuilders 6
40 Practice Questions

A balanced learning style allows individuals to maximize learning in diverse environments. (Shepard p. 59)

136) A physical therapist reviews the medical record of a 26-year-old female recently involved in a motor vehicle
accident. A note in the medical record indicates that the patient uses a halo-vest cervicothoracic orthosis. The MOST
likely medical diagnosis is:
1. spinal fracture
2. acute myofascial pain syndrome
3. traumatic brain injury
4. herniated nucleus pulposus

Answer: 1
Non-Systems
Equipment & Devices; Therapeutic Modalities
A halo-vest cervicothoracic orthosis consists of a rigid halo attached to the skull with four external fixation pins. The device
incorporates four posts that attach to the anterior and posterior of a vest. The device significantly limits approximately 95%
of cervical motion and is most often used in patients with high cervical fractures. (Tan p. 222)

137) A physical therapist attempts to assess a patient's protective sensation in their right foot. The therapist believes the
patient is at significant risk for developing a pressure ulcer due to poor circulation and a long standing history of
diabetes. The optimal method for assessing the patient's sensory integrity is using:
1. Semmes-Weinstein monofilaments
2. wisp of cotton
3. pin with a blunt and sharp end
4. test tubes filled with warm and cool water

Answer: 1
Integumentary System
Examination
Semmes-Weinstein monofilaments are a valid and reliable method to assess light touch sensation. Monofilaments are made
of pieces of nylon that are uniquely calibrated to bend with a precise amount of pressure. By quantifying the ability of the
patient to perceive light touch sensation to exacting standards the therapist can determine the patient's relative risk for
developing a pressure ulcer. (Meyers p. 57)

138) During auscultation of the patient's chest, a physical therapist hears diminished breath sounds. To help confirm
that the lung segment is not fully aerated, the therapist decides to test egophony by:
1. palpating the patient's mediastinum
2. measuring the excursion of the patient's diaphragm
3. auscultating over the lung segment while the patient whispers ?1, 2 ,3?
4. auscultating over the lung segment while the patient speaks the sound ?EEE?

Answer: 4
Cardiac, Vascular, and Pulmonary Systems
Examination
When a pathologic condition is suspected because of the increased or decreased transmission of breath sounds heard during
auscultation, further evaluative measures should be taken. Egophony is demonstrated when a patient says the sound ?EEE?
(long E), but the sound that is auscultated over the chest sounds like ?A? (long A). The change in the quality of the sound
occurs with an increase in the density of lung tissue due to consolidation from atelectasis, secretions or some form of
pathology. When the intensity and clarity of the voiced sounds are heard distinctly through the stethoscope, it indicates the
underlying lung tissue is relatively airless. Additional voice sounds may be used including: bronchophony - ?say 99? and
whispered pectoriloquy - ?whisper 1, 2, 3.? (Hillegass p. 625)

139) A physical therapist applies a custom fabricated hand splint to a patient that sustained a burn to his right wrist and
hand. The splint maintains the wrist in extension; the metacarpophalangeal joints in 75 degrees of flexion; the
proximal and distal interphalangeal joints in flexion; and the thumb in abduction. Based on the supplied description
of the splint, which area would be MOST susceptible to contracture?

Scorebuilders 6
Practice Questions 41

1. wrist
2. metacarpophalangeal joints
3. proximal and distal interphalangeal joints
4. thumb

Answer: 3
Non-Systems
Equipment & Devices; Therapeutic Modalities
The proximal and distal interphalangeal joints (PIPs and DIPs) are susceptible to a flexion contracture when splinted in a
flexed position. Correct positioning of PIPs and DIPs would be in extension. (Goodman - Pathology p. 307)

140) A physical therapist performs a sensory examination on a 38-year-old female and determines that the patient has
no sensation below the L2 dermatome. The patient was admitted through the emergency room 14 days ago
secondary to a motor vehicle accident. What is the MOST likely rationale for the patient's current condition?
1. the patient has a complete transection of the spinal cord
2. the patient has a dorsal column lesion within the spinal cord
3. the patient has a hemisection of the spinal cord
4. the patient has a spinothalamic tract lesion within the spinal cord

Answer: 1
Neuromuscular and Nervous Systems
Foundations for Evaluation, Differential Diagnosis, and Prognosis
Complete transection of the spinal cord would inhibit all sensory information from the ascending pathways. Complete loss
of sensation usually begins one to two levels below the cord lesion since there is overlap in the nerve endings from adjacent
dermatomes surrounding the lesion. (Lundy-Ekman p. 132)

141) A 40-year-old male experiences pain in the left shoulder during shoulder abduction greater than 80 degrees. The
therapist hypothesizes that the patient is experiencing impingement between the head of the humerus and the
acromion as a result of muscle dysfunction. What statement MOST accurately describes the normal role of the
shoulder muscles during shoulder abduction?
1. The scapula upwardly rotates due to action from the levator scapula and serratus anterior muscles. The rotator cuff
depresses the head of the humerus to the lower part of the glenoid fossa as the deltoid abducts the arm.
2. The scapula upwardly rotates due to the action of the upper and lower trapezius and serratus anterior muscles. The
rotator cuff allows only minimal translation of the humeral head and keeps the head centered in the glenoid fossa
as the deltoid abducts the arm.
3. The scapula upwardly rotates due to action of the serratus anterior and teres major muscles. The rotator cuff brings
the head of the humerus anterior in the glenoid fossa
4. The scapula is upwardly rotated 30 degrees by the upper trapezius muscle and the deltoid abducts the arm about
140-150 degrees at the glenohumeral joint. The rotator cuff depresses the head of the humerus approximately one
centimeter in the glenoid fossa.

Answer: 2
Musculoskeletal System
Clinical Application of Foundational Sciences
The upper and lower parts of the trapezius and the serratus anterior muscles form a force couple that upwardly rotates the
scapula as the deltoid abducts the shoulder. The rotator cuff provides a depressive force on the head of the humerus which
allows rotation of the humeral head with minimal translation in the glenoid fossa. (Levangie p. 215)

142) A physical therapist provides patient coverage for a colleague who will be out for the next two weeks. The first
patient the therapist is scheduled to treat has been seen one time in physical therapy and the dictation is not yet in the
patient's chart. The handwritten examination form has only scarce information and MOST of it is illegible. The
referring physician's prescription reads "evaluate and treat - decreased wrist range of motion." The MOST
appropriate therapist action is:
1. reexamine the patient
2. contact the physician's office
3. ask the patient for additional information

Scorebuilders 6
42 Practice Questions

4. reschedule the patient

Answer: 1
Non-Systems
Safety & Professional Roles; Teaching/Learning; Research
The physical therapist must reexamine the patient to attain accurate baseline information for treatment. If the therapist
treated the patient without the requisite information it could be considered professional negligence. (Scott - Promoting
Legal Awareness p. 33)

143) During the physical therapist's physical examination, the therapist notes that the patient's respiratory rate is 30
breaths per minute. The FIRST breathing exercise the therapist should teach the patient is:
1. active cycle of breathing
2. segmental breathing
3. pursed-lip breathing
4. inspiratory muscle training

Answer: 3
Cardiac, Vascular, and Pulmonary Systems
Interventions
Pursed-lip breathing reduces respiratory rate, increases tidal volume, provides symptomatic relief of dyspnea, and improves
exercise tolerance. The active cycle of breathing/forced expiratory technique is a breathing strategy used to assist in
clearance of mucus secretions. The technique combines controlled diaphragmatic breathing with low-to-mid volume huffs
(forced expirations without closing the glottis). Segmental breathing (a.k.a. localized expansion breathing) is used to try to
preferentially enhance localized lung expansion. Manual counter pressure is provided by the therapist against the patient's
thorax to encourage expansion of that specific area of the thorax in the hope of improving ventilation to a specific part of
the lung. The objective of inspiratory muscle training is to increase strength and endurance of the muscles of respiration. A
handheld breathing training device provides a resistance load to the muscles of inspiration. Different inspiratory muscle
training devices provide either flow resistive breathing or threshold breathing. During flow resistive breathing, the patient
inspires through a mouthpiece and adapter with an adjustable diameter. Decreasing the diameter increases the resistance to
breathing. Threshold breathing devices provide consistent and specific pressure for inspiratory muscle training, regardless
of how quickly or slowly patients breathe. (Hillegass p. 736)

144) A 67-year-old male is referred to physical therapy due to multiple right ankle sprains. The patient reports that he
commonly turns his ankle, especially when walking on uneven surfaces. The patient has full pain free active and
passive range of motion and overpressure is within normal limits. Sensory testing utilizing Semmes-Weinstein
monofilaments reveals that the patient first perceives sensation with the 6.10 monofilament (which exerts 75 grams
of pressure) throughout both of his feet. The MOST likely conclusion is that:
1. the patient's sensation is intact and he may therefore have ligamentous instability at the ankle
2. the patient's sensation is compromised and he may therefore have an injury to the L4 nerve root
3. the patient's motor response is not intact and he may therefore have a nerve injury
4. the patient's sensation is compromised and he may therefore be at risk for reinjury

Answer: 4
Other Systems
Foundations for
Evaluation-°Ìÿîÿõÿ÷þÿûíþ±Ìùÿïÿÿÿë÷¾-ÿïö¬ÛÿÿÿþÿóíÿŸßèÿÿ´÷óûÿÿÿùþï¾òÿïöï÷ÿë÷ÿÿÿÿï÷ûõëÿþ±ÿþÿÿíÿ¾ÿÿÿ÷¿ñÿþý«û
ïÿëéüÿöÿ±þïºÿÿþûÿ¨þûõ¶µ¾ÿ¿«ÿïÿïÿûþóÿÿïþ-ÿ÷øëïÿ«÷úÿÿþû¶»±«ÿÿûïó-ïîÿüöýó÷ýöý¿±ëÿüíÿóÿ÷ó¸ÿ¾ýÿÿÿ¾ÿ÷ÿöóÿ÷ÿïÿû
öíºÿÿÿûùüùÿÿòõíÿÿþï¯íþ¯íþÿûÿÿû÷þ-þï÷ï«ÿÿþ«öõùÿþýÿû¾åöúõ¼ðëýÿýþÿ¯÷ºÿïÿûÿþõ«ÿëþÿí¾ÿöÿïÿýùïþ·ÿë÷úÿïþ÷óÿúõ
óûýþÿ¶ëÿ½ÿýôúûîÿîÿÿÿÿí¾ýûÿûÿû¼íþþ¬þÿï÷ÿÿïÿÿ¾ûö¶ûóÿìïûÿÿÿ½öÿñõºöÿÿ³ýÿ÷÷ÿÿÿÿÿýÿ·óþÿÿÿõÿÿ÷ó¬ëÿïýïÿ«÷úÿ¯ÿï
ÿþõ«ÿûõÿýþõë÷õ®ºßëþûö÷±úóþ÷÷°ïÿ«ÿÿÿïÿï¶ûó«ÿëôÿíþ¬ïÿþýûÿ-þíïûþÿ¬¾÷üÿóøÿé÷ºñÿ«ÿþÿïÿ÷õÿïî½üþûþ¬öû÷¬ýÿõïö±ë
ÿÿøÿnent is also intact. (Meyers p. 58)
ßèÿÿ´÷óûÿÿÿùþï¾òÿïöï÷ÿë÷ÿÿÿÿï÷ûõëÿþ±ÿþÿÿíÿ¾ÿÿÿ÷¿ñÿþý«ûïÿëéüÿöÿ±þïºÿÿþûÿ¨þûõ¶µ¾ÿ¿«ÿïÿïÿûþóÿÿïþ-ÿ÷øëïÿ«÷úÿ
ÿþû¶»±«ÿÿûïó-ïîÿüöýó÷ýöý¿±ëÿüíÿóÿ÷ó¸ÿ¾ýÿÿÿ¾ÿ÷ÿöóÿ÷ÿïÿûöíºÿÿÿûùüùÿÿòõíÿÿþï¯íþ¯íþÿûÿÿû÷þ-þï÷ï«ÿÿþ«öõùÿþý
ÿû¾åöúõ¼ðëýÿýþÿ¯÷ºÿïÿûÿþõ«ÿëþÿí¾ÿöÿïÿýùïþ·ÿë÷úÿïþ÷óÿúõóûýþÿ¶ëÿ½ÿýôúûîÿîÿÿÿÿí¾ýûÿûÿû¼íþþ¬þÿï÷ÿÿïÿÿ¾ûö¶
ûóÿìïûÿÿÿ½öÿñõºöÿÿ³ýÿ÷÷ÿÿÿÿÿýÿ·óþÿÿÿõÿÿ÷ó¬ëÿïýïÿ«÷úÿ¯ÿïÿþõ«ÿûõÿýþõë÷õ®ºßëþûö÷±úóþ÷÷°ïÿ«ÿÿÿïÿï¶ûó«ÿëôÿíþ
¬ïÿþýûÿ-þíïûþÿ¬¾÷üÿóøÿé÷ºñÿ«ÿþÿïÿ÷õÿïî½üþûþ¬öû÷¬ýÿõïö±ëÿÿøÿnent is also intact. (Meyers p. 58)

Scorebuilders 6
Practice Questions 43

145) A physical therapist employed in a rehabilitation hospital has significant difficulty motivating a patient to
participate in a home exercise program. The patient is a 52-year-old male diagnosed with multiple sclerosis. The
MOST significant area that the therapist may have overlooked is:
1. using vocabulary appropriate to the patient's level
2. understanding the patient's perspective on his condition
3. involving the family in decision making
4. engaging the patient in a dialogue about discharge

Answer: 2
Non-Systems
Safety & Professional Roles; Teaching/Learning; Research
A therapist is most likely to achieve patient cooperation or adherence to a program when the therapist attempts to
understand the patient's perspective about their condition and its impact on rehabilitation. If a therapist understands the
patient's interpretation of his disease, the therapist can educate and positively affect the patient's desire for cooperation.
(Shepard p. 326)

146) A physical therapist evaluates an eleven-month-old infant recently referred to physical therapy. The therapist
observes that the infant places her hands at shoulder level during ambulation in order to improve stability and
balance. This occurs because:
1. righting reactions are not yet mature
2. protective extension is not yet present
3. equilibrium reactions are still developing
4. tonic labyrinthine reflex is not yet integrated

Answer: 3
Neuromuscular and Nervous Systems
Foundations for Evaluation, Differential Diagnosis, and Prognosis
Equilibrium reactions continue to lag behind the achievement of motor milestones. A child must experience walking and
falling before Eq5i,ibr)u
Reactions develop for this milestone. This results in the use of other postural control mechanisms until the equilibrium
reactions develop. (Campbell - Decision Making p. 40)

147) A physical therapist measures a patient's two-point discrimination at several different areas of the body using a
caliper and a ruler. Which area would the therapist expect to produce the SMALLEST measured value?
1. cheek
2. lateral to the umbilicus
3. medial forearm
4. medial leg

Answer: 1
Neuromuscular and Nervous Systems
Examination
Two-point discrimination is a testing procedure that quantifies the smallest distance between two stimuli where the patient
is still able to identify two distinct points. The distance between the two points is measured with a ruler. Areas of the body
with relatively low two-point discrimination values are in the hands and the face. (O'Sullivan p. 146)

148) A patient is in the final week of an outpatient cardiac rehabilitation program. After completing a 5-minute warm-
up, the patient begins to exercise on the bicycle ergometer. After 5 minutes on the bicycle ergometer, the physical
therapist notes the patient's heart rate is below the prescribed target heart rate zone for exercise. What action should
the therapist take?
1. discontinue the exercise program until the patient has a new graded exercise test to define a new target heart rate
zone
2. continue to have the patient exercise at the current workload while monitoring heart rate, blood pressure, and rate
of perceived exertion
3. increase the workload and continue to monitor heart rate, blood pressure, and rate of perceived exertion
4. reduce the workload and continue to monitor heart rate, blood pressure, and rate of perceived exertio

Scorebuilders 6
44 Practice Questions

Answer: 3
Cardiac, Vascular, and Pulmonary Systems
Interventions
To achieve optimal benefits from aerobic exercise training, the goal should be to maintain an average heart rate close to the
midpoint of the prescribed heart rate zone. A heart rate below the prescribed heart rate zone indicates the patient is not
working hard enough and needs to increase the workload in order to raise the heart rate to the recommended level.
(American College of Sports Medicine p. 144)

149) A 23-year-old male sustains serious burns to over 30% of his body in a house fire. The burns range from
superficial partial-thickness to full-thickness and encompass the majority of the patient's lower extremities. The
MOST appropriate therapeutic position for the patient is:
1. supine with the knees extended and the toes pointing toward the ceiling
2. prone with a pillow placed on the dorsum of the foot and ankle
3. sidelying with the hips and knees slightly flexed using pillows between the legs
4. hooklying with a pillow placed between the knees

Answer: 1
Integumentary System
Interventions
Positioning the patient in supine with the knees extended and the toes pointing toward the ceiling maintains the hips, knees,
and ankles in an optimal position and therefore reduces the likelihood of a lower extremity contracture. (Goodman -
Pathology p. 307)

150) A 52-year-old male is referred to physical therapy after being diagnosed with multiple sclerosis. During the
examination the therapist places a key in the patient's hand. Without looking at the object, the patient is asked to
identify it by touch. This method of testing assesses:
1. directional cutaneous kinesthesia
2. graphesthesia
3. proprioception
4. stereognosis

Answer: 4
Neuromuscular and Nervous Systems
Examination
Stereognosis is the ability to identify an object by touch alone. During testing the patient's vision should be shielded from
the object. There may be damage to the contralateral parietal cortex or adjacent white matter within the brain if the patient
has intact sensation, but cannot identify the object. (Lundy-Ekman p. 127)

151) A physical therapist works on abdominal strengthening exercises with a 38-year-old male with low back
dysfunction. The patient tends to hold his breath during the exercises and as a result the therapist teaches the patient
to breathe during exercise in order to prevent the Valsalva maneuver. Why should the Valsalva maneuver be
avoided?
1. There is a decrease in the intra-thoracic pressure, an increase in the intra-abdominal pressure, and a decrease in
blood pressure
2. There is an increase in the intra-abdominal and intra-thoracic pressures which causes a temporary increase in
blood pressure.
3. There is a decrease in the intra-thoracic and intra-abdominal pressure.
4. There is an increase in the intra-thoracic pressure which puts too much pressure directly on the heart

Answer: 2
Other Systems
Interventions
The Valsalva maneuver should be avoided to eliminate the possibility of a sudden increase of blood pressure. (Kisner p. 98)

Scorebuilders 6
Practice Questions 45

152) A physical therapist has the habit of interrupting and trying to anticipate what other persons, including patients, are
going to say. This habit is interfering with his interpersonal relationships at work and with patients' confidence in the
therapist's ability to care for them. If the therapist plans to implement active listening he might benefit from all of the
following EXCEPT:
1. attend closely to what the other person is saying
2. generate his next statement from what the patient or colleague has said
3. ask closed-ended questions
4. allow the patient or colleague to finish before responding

Answer: 3
Non-Systems
Safety & Professional Roles; Teaching/Learning; Research
If a therapist asks closed-ended questions there is no opportunity for active listening. Yes or no questions do not facilitate
dialogue. All other choices reflect behaviors that are supportive of active listening. (Goodman - Differential Diagnosis p.
38)

153) A physical therapist administers cryotherapy on a patient rehabilitating from arthroscopic knee surgery. Which of
the following physiological effects would the therapist MOST anticipate based on the identified therapeutic agent?
1. increased rate of cell metabolism
2. increased pain threshold
3. increased nerve conduction velocity
4. increased local blood flow

Answer: 2
Non-Systems
Equipment & Devices; Therapeutic Modalities
Cryotherapy increases the pain threshold by inducing analgesia, however decreases the rate of cell metabolism, nerve
conduction velocity, and local blood flow. (Cameron p. 140)

154) A physical therapist examines a four-year-old child with spina bifida at the S1 level. The patient's primary goal is
to ambulate independently. Which of the following would be the MOST appropriate device to consider for this
child?
1. reciprocating gait orthosis
2. knee-ankle-foot orthoses
3. ankle-foot orthoses
4. parapodium

Answer: 3
Non-Systems
Equipment & Devices; Therapeutic Modalities
A child with S1 spina bifida should have only minimal lower extremity muscle involvement with the possible exception of
the ankle evertors and/or plantar flexors. The patient may therefore benefit from the stability offered at the ankle by the
ankle-foot-orthosis. The remaining options provide more support than would be necessary given the child's likely
impairments. (Campbell p. 223)

155) A physical therapist attempts to provide each patient with a home exercise program prior to discharge. A home
exercise program is often a critical component of a patient care plan because:
1. treatment sessions may not occur every day
2. the amount of practice and the amount learned are directly related
3. the amount of practice and the amount learned are inversely related
4. patients participating in home exercise programs require less physical therapy services

Answer: 2
Non-Systems
Safety & Professional Roles; Teaching/Learning; Research

Scorebuilders 6
46 Practice Questions

Research has indicated that the amount of practice directly relates to the amount that is learned by a patient. Home exercise
programs are critical so that patients can continue to practice outside of physical therapy sessions in order to maximize the
long-term functional outcomes. (Shepard p. 403)

156) Under the Individuals with Disabilities Education Act (IDEA), children with disabilities have the right to receive
physical therapy services in the public school setting. Policies mandate that in order to receive these services:
1. the child must be severely impaired
2. the services must support the child's educational success
3. the parents must agree to allow their insurance to be billed for the costs
4. the services must address a medical or rehabilitative goal

Answer: 2
Non-Systems
Safety & Professional Roles; Teaching/Learning; Research
The IDEA stipulates that children are entitled to physical therapy within the school system as long as the goals are related
to educational outcomes. (Scott-Foundations p. 162)

157) A physical therapist employed in an outpatient clinic attends a three day continuing education course emphasizing
advanced manual skills. After completing the course the therapist decides it would be beneficial to disseminate the
information to some of the therapists at his clinic. The MOST appropriate therapist action is:
1. schedule an inservice for the physical therapy staff
2. offer hands on training sessions for interested physical therapy staff
3. encourage the clinic manager to make funding available to send interested physical therapy staff to the course
4. make the course materials available to physical therapy staff

Answer: 2
Non-Systems
Safety & Professional Roles; Teaching/Learning; Research
Hands on training sessions for the physical therapy staff would allow the physical therapist to have the best opportunity to
introduce the advanced manual skills to appropriate staff members. Advanced activities, particularly hands on techniques,
are often best delivered in small groups with participants who already possess specific prerequisite skills in the given area.
Although an inservice would allow the physical therapist to introduce the material it is unlikely the topic would be
appropriate for the entire physical therapy staff and it would be even less likely that meaningful practice opportunities could
be integrated into the larger group session. (Shepard p. 85)

158) A physical therapist takes a new job working in an outpatient private practice. After working on the job for three
weeks the therapist concludes the physical therapist who owns the clinic routinely bills patients for services they do
not receive and on several occasions has modified other therapists' treatment records in an attempt to support the
fraudulent billing activities. The MOST appropriate agency for the physical therapist to report the information to is
the:
1. state licensing agency
2. Federation of State Boards of Physical Therapy
3. American Physical Therapy Association
4. Center for Medicare and Medicaid Services

Answer: 1
Non-Systems
Safety & Professional Roles; Teaching/Learning; Research
The physical therapist has an obligation to report the physical therapist's actions to the state licensing agency. The state
licensing agency is the only available organization that has the power to take definitive action including the possibility of
revoking the therapist's license to practice. (Guide for Professional Conduct)

159) A physical therapist examines the skin of a 42-year-old patient diagnosed with multiple sclerosis. The therapist
identifies a pressure ulcer approximately two centimeters in diameter near the patent's left ischial tuberosity. The
ulcer looks like an abrasion and appears to involve the entire epidermis. The ulcer is BEST classified as:

Scorebuilders 6
Practice Questions 47

1. stage I
2. stage II
3. stage III
4. stage IV

Answer: 2
Integumentary System
Examination
A stage II ulcer is characterized by partial-thickness skin loss involving the epidermis and potentially the dermis. The ulcer
is usually superficial and may present as an abrasion, blister or crater. (Goodman - Pathology p. 308)

160) A physical therapist treats a patient diagnosed with right CVA, left hemiplegia. During the session the therapist
instructs the patient to reach for an object with both hands. Which area of the brain is ultimately responsible for
transmitting the message to move the upper extremities toward the object?
1. association areas of the cerebral cortex in the central sulcus
2. Broca's area in the lateral sulcus
3. primary somatosensory area in the cerebral cortex
4. primary motor cortex in the precentral gyrus

Answer: 4
Neuromuscular and Nervous Systems
Clinical Application of Foundational Sciences
The primary motor cortex is located in the precentral gyrus anterior to the central sulcus. The function of the primary motor
cortex is voluntary controlled movement. There are three motor planning areas (supplementary motor area, premotor area,
and Broca's area) next to the primary motor cortex that receive and process information to assist with movement. (Lundy -
Ekman p. 394)

161) A 35-year-old male is referred to physical therapy with a diagnosis of a grade II spondylolisthesis of L4-L5. The
physical therapist concludes that the patient would benefit from a lumbar stabilization exercise program. What
muscle groups would BEST stabilize the L4- L5 spinal segment?
1. obliquus externus abdominis, longissimus thoracis (thoracic attachments), psoas major
2. transversus abdominis, iliocostalis lumborum (thoracic attachments), quadratus lumborum
3. quadratus lumborum, lumbar multifidi, longissimus thoracis (lumbar attachments)
4. transversus abdominis, psoas major, longissimus thoracis (lumbar attachments)

Answer: 3
Musculoskeletal System
Interventions
The lumbar multifidi and lumbar attachments of the longissimus thoracis are short deep muscles that will provide the most
segmental stability. The quadratus lumborum will add additional stability in the frontal plane. (Neumann p. 329)

162) A physical therapist prepares to administer an iontophoresis treatment to a patient with myositis ossificans in the
right mid-quadriceps region. Which treatment description would be the MOST appropriate?
1. driving dexamethasone with a high volt pulsed current unit using the negative pole
2. driving dexamethasone with a high volt pulsed current unit using the positive pole
3. driving acetic acid with a direct current unit using the negative pole
4. driving acetic acid with a direct current unit using the positive pole

Answer: 3
Non-Systems
Equipment & Devices; Therapeutic Modalities
Iontophoresis utilizes direct current to move charged particles (ions) into the tissue for therapeutic purposes. Acetic acid has
a negative polarity and therefore must be driven by the negative pole. Acetic acid can be beneficial to break up calcific
deposits. (Cameron p. 235)

Scorebuilders 6
48 Practice Questions

163) After palpating several peripheral pulse sites a physical therapist concludes that a patient has a strong pulse at the
popliteal artery, but a weak pulse at the dorsalis pedis artery. This finding suggests that during exercise the patient
may experience:
1. orthopnea
2. deep vein thrombophlebitis
3. intermittent claudication
4. venous thrombosis

Answer: 3
Cardiac, Vascular, and Pulmonary Systems
Foundations for Evaluation, Differential Diagnosis, and Prognosis
Palpation of the arterial pulses can assess perfusion of the extremities. Patients with diabetes or peripheral vascular disease
often have diminished pulses, particularly in the hands and feet. The patient may experience symptoms of ischemia, such as
intermittent claudication, when blood flow is not adequate to meet the demands of peripheral tissues (i.e. during activity).
(Hillegass p. 633)

164) A physical therapist examines a 37-year-old female referred to physical therapy with an abdominal strain. The
patient reports an insidious onset of pain that is worse after eating certain foods. The patient describes the pain as a
dull ache located slightly to the right of the center of the back at the level of the inferior border of the scapula. The
patient works at a factory and spends several hours a day bent over a conveyer belt. An examination reveals adequate
spinal mobility with only minor muscle spasm and no tenderness is noted. The patient's clinical presentation is
MOST consistent with dysfunction of the:
1. pancreas
2. liver
3. gallbladder
4. kidneys

Answer: 3
Other Systems
Foundations for Evaluation, Differential Diagnosis, and Prognosis
A common symptom associated with cholecystitis (gallbladder inflammation) is right shoulder pain and pain between the
scapulae. Recognizing the patient has good spinal mobility with only minimal muscle spasm rules out musculoskeletal
pathology. (Goodman -Differential Diagnosis p. 273)

165) A physical therapist employed in an inpatient rehabilitation hospital treats a patient diagnosed with a traumatic
brain injury. The patient has difficulty with rolling secondary to the presence of a primitive reflex. When the patient
attempts to initiate rolling through turning his head, his arms move into a "fencing" posture with the upper extremity
extended on the side to which the head is turned and the other extremity flexed. This posture is associated with
which of the following reflexes?
1. symmetrical tonic neck reflex
2. asymmetrical tonic neck reflex
3. symmetrical tonic labyrinthine
4. neck righting action on the body

Answer: 2
Neuromuscular and Nervous Systems
Interventions
The asymmetric tonic neck reflex is stimulated when a patient's head is rotated to one side. This action potentially inhibits
an infant from rolling since it is difficult to move the body over the extended upper extremity. (Shumway-Cook p. 195)
A

166) Under the auspices of the Americans with Disabilities Act, which of the following would not be considered a
reasonable accommodation for a physical therapist working in a busy inpatient neurological rehabilitation program?
1. a full-time aide to assist with note taking and writing tasks
2. an aide available to assist with transfers
3. an adapted work station
4. a flexible or modified work schedule

Scorebuilders 6
Practice Questions 49

Answer: 1
Non-Systems
Safety & Professional Roles; Teaching/Learning; Research
A reasonable accommodation must not place undue hardship on the work place or fundamentally change the nature of the
job. Hiring an aide for a physical therapist's full-time personal use is not typically considered a reasonable accommodation.
(Scott-Promoting Legal Awareness p. 195)

167) A physical therapist observes the gait of a 59-year-old male. The patient has a functionally fused ankle due to
arthritic changes precipitated from a severe fracture sustained in a motor vehicle accident ten years ago. Which of the
following shoe modifications would be the MOST beneficial to assist the patient?
1. cushioned heel
2. metatarsal bar
3. rocker bar
4. heel lift

Answer: 3
Non-Systems
Equipment & Devices; Therapeutic Modalities
A rocker bar is a convex strip placed across the sole of a shoe immediately posterior to the metatarsal heads. The device
would partially compensate for the limitation in ankle range of motion by shifting the rollover point posterior to the
metatarsal heads and transferring load to the metatarsal shafts. (Tan p. 185)

168) A physical therapist examines the lower extremity of a patient with a transtibial amputation. During the
examination the therapist determines the patient has weak (2/5) adductors and normal (5/5) abductors. Based on the
patient's clinical presentation, what position might the lower extremity tend to favor in standing?
1. adduction
2. abduction
3. adduction and lateral rotation
4. abduction and medial rotation

Answer: 2
Musculoskeletal System
Foundations for Evaluation, Differential Diagnosis, and Prognosis
Weakness in the adductors in relation to the abductors would tend to make the lower extremity assume an abducted
position. A significant imbalance between oppositional muscle groups typically create a decided bias toward the stronger
muscle group. There would not likely be a decided bias toward lateral or medial rotation since selected abductors can assist
with both motions. For example the anterior fibers of the gluteus medius assist with medial rotation while the posterior
fibers assist with lateral rotation. (Kendall p. 221)

169) A physical therapist examines a patient diagnosed with suspected arterial occlusive disease. The therapist
identifies a number of findings consistent with the diagnosis including absent femoral pulse, dependent rubor, and
intermittent claudication in the buttocks, hamstrings, and calf muscles. The MOST likely site of occlusion is the:
1. iliac artery
2. femoral artery
3. popliteal artery
4. tibial artery

Answer: 1
Cardiac, Vascular, and Pulmonary Systems
Foundations for Evaluation, Differential Diagnosis, and Prognosis
Occlusive disease of the iliac artery would likely result in significant findings throughout the lower extremities. Intermittent
claudication extending into the buttock area and an absent femoral pulse combine to make the iliac artery the most probable
site for occlusion. (Goodman - Pathology p. 452)

Scorebuilders 6
50 Practice Questions

170) During an examination a female patient expresses to the physical therapist that she has difficulty shrugging her
shoulders and turning her head due to a nerve injury. Based on the patient's comment, the cranial nerve MOST likely
affected is:
1. vestibulocochlear
2. vagus
3. spinal accessory
4. hypoglossal

Answer: 3
Neuromuscular and Nervous Systems
Clinical Application of Foundational Sciences
Shoulder shrugging (scapular elevation) and turning of the head are primarily controlled by the sternocleidomastoids and
the upper trapezius muscles. These muscles are innervated by cranial nerve XI, also known as the spinal accessory nerve.
(Lundy-Ekman p. 323)

171) A 65-year-old female was referred to physical therapy for mobilization of her left wrist. The patient sustained a
Colles' fracture eight weeks ago and although the fracture is well healed the patient continues to experiences
significant wrist stiffness. What technique would be the MOST appropriate to help improve extension at the
radiocarpal joint?
1. glide the lunate and capitate anteriorly in relation to the radius
2. glide the triquetrum and lunate anteriorly in relation to the head of the ulna
3. glide the lunate and scaphoid anteriorly in relation to the radius
4. glide the scaphoid and capitate posteriorly in relation to the radius

Answer: 3
Musculoskeletal System
Interventions
The convex joint surface normally glides in the opposite direction of the angular movement of the bone. The lunate and
scaphoid articulate with the concave joint surface of the radius. (Kisner p. 241)

172) A physical therapist assists a member of the nursing staff to measure the ankle-brachial index (ABI) of a patient
recently admitted to the hospital due to complications from diabetes. After conducting the measurement the ABI is
determined to be 1.2. The MOST likely interpretation of the test is:
1. the patient has moderate arterial insufficiency
2. the patient has arterial calcification
3. the patient has superior peripheral tissue perfusion
4. the patient's ABI is within normal limits

Answer: 2
Cardiac, Vascular, and Pulmonary Systems
Examination
Arterial calcification results in arterial walls becoming extremely stiff which therefore requires more pressure to compress
and ultimately collapse the vessel. As a result this condition will tend to produce artificially high ABI values. In instances
of arterial calcification the ABI is not a valid indicator of peripheral tissue perfusion. (Meyers p. 211)

173) A physical therapist assesses the heart rate and SaO2 of a 42-year-old male diagnosed with chronic obstructive
pulmonary disease during an exercise session. The therapist notes an increase in the patient's resting heart rate of 15
beats per minute and a decrease in SaO2 below the minimum acceptable saturation level of 90%. What action should
the therapist take?
1. activate the emergency medical system
2. select an alternate exercise activity
3. stop activity and have the patient take his prescribed beta-blocker medication
4. consult with the patient's physician about whether this patient could use supplemental oxygen during activity

Answer: 4
Cardiac, Vascular, and Pulmonary Systems

Scorebuilders 6
Practice Questions 51

Clinical Application of Foundational Sciences


Typically, the SaO2 level should remain between 90 and 94% for patients with respiratory pathology. If the SaO 2 drops
below the minimum threshold during activity, supplemental oxygen may be used to keep the saturation level in the safe
range. Since increasing O2 may reduce hypoxic drive in patients with COPD, the physician must determine a minimal
acceptable saturation level and provide titration guidelines should the patient's SaO 2 level decrease with activity. (Hillegass
p. 665)

174) A physical therapist uses the Braden Scale to quantify a patient's risk for developing a pressure ulcer. After
administering the test the therapist determines that the patient scored a ten. Based on the results, the patient's risk for
developing a pressure ulcer would be BEST classified as:
1. not at risk
2. moderately low
3. moderately high
4. high risk

Answer: 4
Integumentary System
Examination
Studies have indicated that individuals scoring below a 12 on the Braden Scale are at high risk (>80%) for developing a
pressure ulcer. (Goodman - Pathology p. 309)

175) A physical therapist is working on wheelchair to bed transfers with a patient who is status post brain injury. The
physical therapist first assists the patient with locking the wheelchair brakes. Following this, the therapist attempts to
direct the patient to complete the rest of the transfer, yet the patient repeatedly attempts to lock the wheelchair
brakes. While doing so, the patient reports "I am trying to move over to the bed." The patient is demonstrating which
of the following?
1. impaired judgment
2. perseverative behavior
3. a visual field cut
4. impaired procedural memory

Answer: 2
Neuromuscular and Nervous Systems
Foundations for Evaluation, Differential Diagnosis, and Prognosis
In this case, the patient has already locked the wheelchair brakes and does not seem to recognize that he is continuing to
perform the same task repeatedly. This is an example of a phenomenon called perseveration which often occurs in patients
following a brain injury. (O'Sullivan p. 794)

176) A 58-year-old male with diabetes mellitus and a long history of smoking is referred to physical therapy. The
patient's physician has determined that a transtibial amputation is now indicated. During the post-surgical hospital
stay, the patient indicates that he is MOST comfortable sitting upright in a wheelchair with his hips and knees at a 90
degree angle. The MOST serious concern that could complicate the patient's rehabilitation potential is:
1. the position can lead to shortening of the hamstrings and rectus femoris muscles
2. failure to be active will reduce the patient's cardiovascular endurance
3. it is difficult to work on strengthening exercises from this position
4. it may indicate that the patient lacks motivation for prosthetic training

Answer: 1
Other Systems
Interventions
Full range of motion at the hip and knee are needed to ensure efficient and effective use of a prosthetic limb. (Seymour p.
145)

Scorebuilders 6
52 Practice Questions

177) A physical therapist examines a 32-year-old male referred to physical therapy with significant lower extremity
weakness. The patient presently uses double upright metal knee-ankle-foot orthoses for ambulation. The devices
incorporate a locked knee joint using drop ring locks. Which objective finding would be the LEAST likely during the
examination?
1. diminished quadriceps strength
2. diminished knee extension range of motion
3. lower extremity spasticity
4. impaired proprioception at the knee and ankle

Answer: 2
Non-Systems
Equipment & Devices; Therapeutic Modalities
Locked knee joints require a patient to possess full active or passive knee extension. Drop ring locks permit the knee joints
to be locked by gravity or manual assistance while the uprights are fully extended. (Tan p. 195)

178) A physical therapist employed in an acute care hospital identifies a recent note in a patient's chart that indicates the
physician ordered an x-ray of the cervical spine using an oblique view (posteroanterior). This view would be ideal to
examine the:
1. articular pillars
2. intervertebral foramina
3. posterior arch of C1
4. spinous process of C2

Answer: 2
Musculoskeletal System
Foundations for Evaluation, Differential Diagnosis, and PrognosisAn x-ray of the cervical spine using an oblique view
(posteroanterior) allows visualization of the open intervertebral foramina and cervical pedicles. (Greathouse p. 350)

179) A physical therapist examines a patient recently admitted to the hospital with chronic bronchitis. While examining
the patient's lung sounds the therapist identifies a distinct wheezing sound associated with inspiration. The MOST
probable explanation is
1. hyperinflation of the lungs
2. fluid accumulation in the pleural space
3. partial obstruction of the larynx or trachea
4. excessive mucus production in the bronchi

Answer: 3
Cardiac, Vascular, and Pulmonary Systems
Foundations for Evaluation, Differential Diagnosis, and Prognosis
The term stridor describes a wheeze that is associated with inspiration. This condition requires immediate medical attention
since it is often associated with a partial obstruction of the larynx or trachea. (Bickley p. 241)

180) A physical therapist observes a patient rehabilitating from a rotator cuff tear abduct his shoulder while holding a
one pound weight. In this activity the middle deltoid is working as a:
1. first class lever
2. second class lever
3. third class lever
4. the example does not describe a first, second or third class lever

Answer: 3
Musculoskeletal System
Clinical Application of Foundational Sciences
In a third class lever, the force component lies between the axis (shoulder joint) and the resistance (gravity). In this case,
gravity is considered to be the middle of the mass being lifted (usually located approximately half way across the part being
moved). (Norkin p. 30)

Scorebuilders 6
Practice Questions 53

181) A 42-year-old male patient has experienced shoulder pain for two months. After completing several goniometric
measurements the therapist determines that with the shoulder abducted to 90 degrees the patient has 90 degrees of
lateral rotation and 70 degrees of medial rotation. Additional testing indicates that the patient has 160 degrees of
shoulder flexion when tested in supine with the knees bent and the feet flat. From this information what would be the
BEST hypothesis to make concerning the patient's shoulder?
1. the patient may have a capsular pattern of restriction
2. the lateral rotators may be tight
3. the patient may have subacromial impingement
4. the latissimus dorsi may be tight

Answer: 4
Musculoskeletal System
Foundations for Evaluation, Differential Diagnosis, and Prognosis
Shoulder flexion range of motion is moderately limited which could be caused by tightness in the latissimus dorsi. Medial
and lateral rotation range of motion is within normal limits and therefore the restricted shoulder motion does not follow a
capsular pattern. (Sahrman p. 212)

182) A physical therapist prepares to initiate mechanical cervical traction to a 210 pound male with degenerative disk
disease of the cervical spine. The therapist's goal is to create joint distraction. Which of the following forces would
be the MOST appropriate to accomplish the therapist's objective?
1. 5 lbs
2. 15 lbs
3. 25 lbs
4. 35 lbs

Answer: 2
Non-Systems
Equipment & Devices; Therapeutic Modalities
Seven percent of the patient's body weight is usually adequate to create joint distraction within the cervical spine. Seven
percent of the body weight of a 210 pound patient is 14.7 lbs. (Cameron p. 329)

183) A physical therapist performs postural drainage activities emphasizing vibration on a patient with cystic fibrosis.
The therapist selects the interventions to prevent complications of obstruction and infection within the airways. The
therapist should perform vibration:
1. during the patient's inhalation
2. during the patient's exhalation
3. before percussion
4. after percussion

Answer: 2
Cardiac, Vascular, and Pulmonary Systems
Interventions
At the end of a deep inspiration, the therapist exerts mild pressure on the patient's chest wall and gently oscillates it through
the end of expiration. The intervention may be a useful alternative to percussion in patients with chest wall discomfort or
pain. (Hillegass p. 651)

184) A 40-year-old female is referred to a physical therapy clinic for range of motion exercises. The patient was
severely burned one year ago over the right anterior elbow. Upon visual inspection the therapist notes severe scar
tissue, which extends several inches above and below the anterior elbow. The scarring appears to extend beyond the
original borders of the burn. The MOST likely explanation for this occurrence is:
1. inadequate fibroplasia with adequate contraction
2. collagen synthesis exceeded collagen lysis
3. collagen lysis exceeded collagen synthesis
4. excessive fibroplasia with inadequate contraction

Answer: 2

Scorebuilders 6
54 Practice Questions

Integumentary System
Clinical Application of Foundational Sciences
A keloid scar occurs when collagen synthesis exceeds collagen lysis. A keloid scar will extend beyond the original borders
of the wound while hypertrophic scarring is usually contained within the wound's boundaries. (Michlovitz p. 13)

185) A physical therapist provides an educational session for a group of individuals newly diagnosed with diabetes
mellitus. The goal of the session is to reduce the participant's risk of developing diabetic foot ulcers. This is an
example of:
1. early prevention
2. primary prevention
3. secondary prevention
4. tertiary prevention

Answer: 2
Other Systems
Interventions
Primary prevention is defined as preventing a condition in a susceptible or potentially susceptible population through
measures such as health promotion efforts. (Guide to Physical Therapist Practice p. 41)

186) A physical therapist administers air to a patient's lungs using a bag-valve-mask resuscitation device. What is the
MOST IMMEDIATE method to determine if the air is reaching its target area?
1. identify the extent of pupil dilation
2. determine if the patient's chest is rising
3. examine the color of the patient's skin
4. determine if air is escaping from the patient's nostrils

Answer: 2
Non-Systems
Safety & Professional Roles; Teaching/Learning; Research
Observing the patient's chest rise is an indication that air is reaching the patient's lungs. The bag-valve-mask resuscitator
device consists of a self-inflating bag, a one way valve, and a mask. The device reduces the risk of disease transmission and
increases the level of oxygen being delivered to the patient. (American Red Cross p. 22)

187) A 50-year-old male reporting for a physical therapy visit proudly reports that his cholesterol level was recorded as
160 mg/dL. Assuming the patient has no known history of cardiac risk factors, what would be the recommended
interval for another formal cholesterol test?
1. six months
2. one year
3. two years
4. five years

Answer: 4
Cardiac, Vascular, and Pulmonary Systems
Foundations for Evaluation, Differential Diagnosis, and Prognosis
The primary purpose of cholesterol screening is to determine an individual's risk of heart disease. The target population for
cholesterol screening is males and females between 45 and 65 years of age. Formal cholesterol screening once every five
years is recommended unless there are other known risk factors for coronary artery disease. (Edelman p. 236)

188) A physical therapist is exercising a patient with emphysema in the patient's home when the patient becomes short
of breath. The patient's oxygen saturation measured with a pulse oximeter is 85%. The therapist believes the patient
would benefit from using supplemental oxygen during exercise. Who can prescribe oxygen for the patient?
1. physical therapist
2. physician
3. nurse
4. respiratory therapist

Scorebuilders 6
Practice Questions 55

Answer: 2
Non-Systems
Safety & Professional Roles; Teaching/Learning; Research
Oxygen is considered a drug when it is breathed in concentrations higher than that found in the atmosphere. Thus,
supplemental oxygen must be prescribed by a physician. The prescription is based on the patient's diagnosis, age, degree of
hypoxemia at rest, during exercise, and during sleep. The oxygen prescription should include a variety of flow rates
according to the patient's activity level. (Brannon p. 327)

189) A physical therapist completes the alar ligament test on a patient as part of a cervical examination. After
completing the test, the physical therapist concludes that the test is positive since there was ample lateral flexion
available without any evidence of an end-feel. The MOST appropriate therapist action is to:
1. continue with the examination and subsequent treatment
2. continue with the examination and subsequent treatment, however avoid direct cervical mobilization techniques
3. continue with the examination and subsequent treatment, however send the referring physician a copy of the
examination results
4. discontinue the examination and contact the referring physician

Answer: 4
Musculoskeletal System
Examination
A positive alar ligament test may be indicative of a serious medical condition such as a fracture of the dens. As a result, the
finding warrants immediate contact with the referring physician. (Magee p. 160)

190) A physical therapist examines a 75-year-old female status post severe right CVA. The patient was admitted
through the emergency room six days ago and the orders are for a bedside evaluation. The therapist attempts to
assess the patient's state of consciousness during the examination. The patient is unresponsive to a variety of verbal
and other types of stimuli, however, is aroused after the therapist pinches her Achilles tendon. The patient's present
state of consciousness is BEST documented as:
1. a coma
2. a catatonic state
3. a stupor
4. a vegetative state

Answer: 3
Neuromuscular and Nervous Systems
Examination
A stupor is characterized by being aroused only by strong stimuli such as a strong pinch. (Lundy-Ekman p. 345)

191) A 35-year-old female is referred to physical therapy for rehabilitation after sustaining an injury to her left knee and
lower leg in an automobile accident six weeks ago. The patient had ligament damage in her knee as well as extensive
contusions in the calf. Sensory evaluation revealed some loss of superficial tactile sensation over the lateroposterior
aspect of the lower leg and heel. What peripheral nerve would MOST likely be involved?
1. sural nerve
2. lateral plantar nerve
3. common fibular (peroneal) nerve
4. saphenous nerve

Answer: 1
Neuromuscular and Nervous Systems
Clinical Application of Foundational Sciences
The sural nerve supplies the skin on the lateral-posterior side of the lower leg and heel. (Saidoff p. 333)

192) A physical therapist treats a patient status post biceps tendon repair. The patient complains of continued difficulty
lowering a five pound bag of flour from the top shelf of her kitchen cabinet to the countertop. In this activity, the
biceps brachii would be performing a/an:

Scorebuilders 6
56 Practice Questions

1. cocontraction
2. concentric contraction
3. eccentric contraction
4. isometric contraction

Answer: 3
Musculoskeletal System
Clinical Application of Foundational Sciences
Eccentric contractions are utilized when a muscle is lengthening while controlling movement against resistance. (Pierson p.
121)

193) A physical therapist notices that a patient status post thoracic surgery has significantly altered his breathing
pattern. The therapist hypothesizes that the alteration in breathing may be exacerbated due to pain from a
sternotomy. The MOST appropriate intervention to treat and prevent atelectasis is:
1. pursed-lip breathing
2. inspiratory muscle training
3. incentive spirometry
4. postural drainage

Answer: 3
Other Systems
Interventions
In a pattern of shallow breathing, a gradual alveolar collapse may develop which can lead to atelectasis. Incentive
spirometry should be performed frequently, up to every hour, to treat and prevent atelectasis, especially in patients status
post thoracic and abdominal surgery. (Hillegass p. 529)

194) A physical therapist completes a positional assessment of a patient's pelvis and sacrum. Which objective finding
BEST supports the presence of a right anteriorly rotated innominate?
1. left posterior superior iliac spine superior to the right posterior superior iliac spine
2. right posterior superior iliac spine superior to the left posterior superior iliac spine
3. right anterior superior iliac spine superior to the left anterior superior iliac spine
4. left anterior superior iliac spine inferior to the right anterior superior iliac spine

Answer: 2
Musculoskeletal System
Foundations for Evaluation, Differential Diagnosis, and Prognosis
A right anteriorly rotated innominate examined in standing would result in the right posterior superior iliac spine being
superior to the left posterior superior iliac spine and the right anterior superior iliac spine being inferior to the left anterior
superior iliac spine. (Hertling p. 708)

195) A physical therapist works with a 40-year-old male who is status post stroke. The patient reports having difficulty
clearing his leg when taking a step. When observing his gait, it is clear that the patient advances his leg by hip
hiking. Which of the following activities would be the MOST beneficial to improve the patient's motor control
during swing?
1. stretching of the gastrocnemius
2. marching in place when standing
3. slow rotation of the leg to reduce extensor spasticity
4. prone knee bends

Answer: 2
Neuromuscular and Nervous Systems
Interventions
Studies show that inadequate recruitment of agonists, as opposed to increased muscle tone and activity in the antagonist, is
the most likely contributor to functional loss following a central nervous system lesion. Marching in place when standing
will help to strengthen all muscles involved in the swing phase of gait and therefore is more desirable than prone knee
bends. (Shumway-Cook p. 135)

Scorebuilders 6
Practice Questions 57

196) A patient rehabilitating from a stroke receives an order for a solid ankle-foot orthosis set in 5 degrees of
dorsiflexion on his left lower extremity. After receiving the orthosis the patient's gait is markedly improved, however
he has developed a blister over the longitudinal arch of the left foot. What is the MOST likely cause of the blister?
1. excessive shear force between the orthosis and the skin
2. excessive compressive force
3. torsion between the ankle-foot orthosis and the ankle joint
4. a bending moment along the plantar surface of the orthosis

Answer: 1
Non-Systems
Equipment & Devices; Therapeutic Modalities
Blisters may be caused by friction between the limb and the orthosis. The physical therapist should make sure that the
patient understands the importance of wearing a sock between the skin and the orthosis. (Seymour p. 47)

197) A physical therapist inexperienced in working with support personnel often fails to utilize physical therapist
assistants since he is unclear of their appropriate role. The MOST appropriate source to consult in order to utilize
physical therapist assistants in an appropriate manner is:
1. Center for Medicare and Medicaid Services
2. state licensing agency
3. Federation of State Boards of Physical Therapy
4. American Physical Therapy Association

Answer: 2
Non-Systems
Safety & Professional Roles; Teaching/Learning; Research
The role of the physical therapist and physical therapist assistant is determined by practice acts established by individual
state licensing agencies. The American Physical Therapy Association serves as a recommending body related to physical
therapy practice at a national level and therefore does not exercise any formal ability to determine rules and regulations
within a given jurisdiction. As a result the physical therapist should consult the individual state licensing agency within his
state. (Scott-Foundations p. 327)

198) A physical therapist performs a muscle test of the shoulder lateral rotators using a handheld dynamometer. The
MOST appropriate location to position the dynamometer is:
1. over the anterior aspect of the distal humerus just proximal to the elbow
2. over the posterior aspect of the distal humerus just proximal to the elbow
3. over the anterior aspect of the distal forearm just proximal to the wrist
4. over the posterior aspect of the distal forearm just proximal to the wrist

Answer: 4
Musculoskeletal System
Examination
A handheld dynamometer serves as a device to quantify the amount of resistance during muscle testing activities. In order
to test the lateral rotators the therapist would need to place the dynamometer over the posterior aspect of the distal forearm
just proximal to the wrist and offer resistance in the direction of medial rotation. (Kendall p. 281)

199) A physical therapist reviews a graphical representation of selected pulmonary function tests and capacities. Which
two items combine to form functional residual capacity?
1. inspiratory reserve volume and tidal volume
2. vital capacity and residual volume
3. inspiratory reserve volume and expiratory reserve volume
4. expiratory reserve volume and residual volume

Answer: 4
Cardiac, Vascular, and Pulmonary Systems

Scorebuilders 6
58 Practice Questions

Clinical Application of Foundational Sciences


Functional residual capacity is defined as the volume of air remaining in the lungs at resting end-expiratory level.
Functional residual capacity is the sum of expiratory reserve volume and residual volume. (Goodman - Pathology p. 1193)

200) A physical therapist prepares to write a S.O.A.P. note after performing gait training activities with a patient status
post total hip replacement. The statement "I'm getting a little dizzy, I better sit down" should be included in the
section labeled:
1. subjective
2. objective
3. assessment
4. plan

Answer: 1
Non-Systems
Safety & Professional Roles; Teaching/Learning; Research
Statements made by the patient relevant to their condition or treatment should be included in the subjective section of a
S.O.A.P. note. (Quinn p. 127)

Scorebuilders 6

Potrebbero piacerti anche